

Hello,
Dr. Batman
Hello Doctor, Welcome!
Profile

Name: Batman
Email: batman@gotham.com
ETHICS
(Total Questions - 205)Q.1. A 17 year old boy is brought into the Emergency Department unconscious following a road traffic accident. What is the most appropriate action?
Correct Answer : A
As per international ethics guidelines, this case is an emergency. Therefore, we can begin immediately treating the patient without their consent as it is necessary to save their life.
Once they have regained capacity within your care, you can inform them of what you have done and ask for them to make ongoing treatment decisions.
Q.2. A 34 year old woman requests for sterilization. Her last born child has cerebral palsy. Her husband strongly objects to the procedure. She has considered all other modes of contraception but has decided on sterilization. What is the most appropriate action?
Correct Answer : B
In this case stem, there is no indication that the patient’s mental capacity is impaired. Under consent guidelines, “no one else can make a decision on behalf of an adult who has the capacity”.
In addition, it is up to the patient to choice whether to involve their family members or carers in their decision-making process. Therefore, you would proceed with the patient’s wishes assuming that you have thoroughly explained the procedure, its outcomes, and risks, and the patient has been fully informed to her full satisfaction.
Q.3. A 15 year old girl attends the surgery for contraception advice. She is mentally competent and understands the implications of her sexual relationship. What is the most appropriate action?
Correct Answer : C
Under the Fraser criteria, it would be appropriate to prescribe her contraception. You can encourage her to inform her parents, but you should not insist on that as ultimately it is her decision as she is competent. Finding out the age of her sexual partner is important as it would raise red flags if the partner was much older or someone who is in power like a teacher. If this was the case, you would need to consider child protection issues and have the duty to inform relevant authorities.
Breaching confidentiality in a sexually active young person, you can provide contraceptive, abortion, and STI advice and treatment, without parental knowledge or consent.
If the options provide both social services and police, pick social services first as this would be what the majority of GPs would do, and social services would normally carry on by contacting the police.
Q.4. A 55 year old woman has recently been diagnosed with multiple sclerosis and has been started on oral steroids. She is brought to the hospital after having ingested more than 40 tablets of paracetamol 3 hours ago. She is refusing all medical treatment as she wants to end her life. What is the most appropriate action?
Correct Answer : A
The patient has multiple sclerosis. The first thing to do if she is refusing treatment is to evaluate her mental capacity to do so.
Q.5. A 16 year old boy has lower abdominal pain. A clinical suspicion of acute appendicitis was made and the surgical team has decided to go ahead with an appendectomy. The patient is happy to go ahead with the surgery but his parents refuse to sign the consent form as they are opposed to any form of surgery. The procedure has been explained clearly to the patient and his parents and they understand the risk of not going for surgery. What is the most appropriate action?
Correct Answer : A
At 16 it is legally presumed that young people can make decisions about their care. Note that even children under 16 may have the capacity to consent provided they have the maturity to do so. In this case, the parents are refusing treatment that is clearly in the best interests of a child. It is clear that the child has the maturity and ability to understand what is involved and he is of good age to give consent.
The capacity to consent depends more on young people’s ability to understand and weigh up options than on age. When assessing a young person’s capacity to consent, you should bear in mind that:
- At 16 a young person can be presumed to have the capacity to consent
- A young person under 16 may have the capacity to consent, depending on their maturity and ability to understand what is involved.
Respect for young people’s views is important in making decisions about their care. If they refuse treatment, particularly treatment that could save their life or prevent serious deterioration in their health, this presents a challenge that you need to consider carefully. Parents cannot override the competent consent of a young person to treatment that they consider is in their best interests.
But you can rely on parental consent when a child cannot consent. Parents cannot authorize treatment if a competent young person has refused.
You should seek legal advice if you think treatment is in the best interests of a competent young person who refuses. Note that the ability to consent changes if you are under 13. By law children under 13 are considered unable to consent.
Q.6. A 31 year old woman who was found unconscious is brought by ambulance to the Emergency Department. The paramedics state that she was seen next to empty packets of paracetamol. Serum paracetamol levels were requested and the results were plotted above the treatment line. Her blood pressure is 120/80 mmHg. She has a pulse of 80 beats/minute and a respiratory rate of 18 breaths /minute. A venous blood gas was performed which shows a pH of 7.38. She remains unconscious but reacts to pain. What is the most appropriate next step?
Correct Answer : B
The doctors must have consent before carrying out any examination, investigation, or providing treatment. However, if the patient is unconscious and unable to express her wishes, you should treat the patient appropriately as a paracetamol overdose could be fatal.
Q.7. A mother brings her 12 year old son to you requesting surgery for his sticking out ears. She says that he is constantly teased in school because of his ears and wants him to undergo cosmetic surgery for it. The boy says he does not want surgery. What is the most appropriate action?
Correct Answer : C
The topic of consent is extremely important for the exam as well as practice. A person aged 16 and upward is generally considered to have the capacity and can consent to their medical procedures. A child is also able to consent provided they are competent in understanding the situation.
This is called Gillick competence. In the above situation, the best choice is to contact social services and assess the boy’s capacity to make his own decision. Surgery is a major procedure and this child should not be subject to undergo a potentially life-threatening procedure that he doesn’t need just because his caregiver wishes him to.
Q.8. A 13 year old girl presents to the clinic requesting morning pills. She says the condom she used with her 13 year old boyfriend split into two while having intercourse. What is the most appropriate action?
Correct Answer : A
In this case, both the girl and her boyfriend are the same age. There is no need to inform the police about their sexual activity as they are the same age. If the boyfriend was much older, and thereby had a greater disparity in age to the girl, then informing the police and/or social services would be considered.
In this case, you are permitted to provide contraception to the girl without parental knowledge or consent as it is most likely in her best interests to do so.
Q.9. A 34 year old pregnant woman was involved in a road traffic accident and presents to the hospital with circulatory collapse. She is a Jehovah’s Witness and carrying a witnessed written directive refusing blood and all blood products. She is taken to the operating table where the surgical team is finding it difficult to stop the bleeding. Her haemoglobin is 40 g/L. She urgently requires blood transfusion. What is the most appropriate action?
Correct Answer : A
The patient has a witnessed written directive which is valid, and therefore her wishes should be respected even if it results in the death of the patient. Since she has refused all blood and blood products, even platelets may not be infused. The only option is to proceed with fluid resuscitation.
Fetuses do not have rights until they are born. So, being pregnant would not change the decision for a blood transfusion. This is one of the most distressing situations surgeons may find themselves in.
Q.10. An armed robber is injured during an exchange of gunfire with the police and comes to the Emergency Department. He requests your silence. What is the most appropriate action?
Correct Answer : A
Any person arriving at the hospital with a gunshot wound needs to be reported to the police. The reasoning behind this is to ensure the safety of the patient, the medical staff, the patients, and visitors within the hospital, as well as to prevent another attack. It is important to note that the incident of the gunshot wound is to be reported initially and not necessarily the personal details of the armed robber.
Personal information, such as the patient’s name and address, should not usually be disclosed in the initial contact with the police.
The police will respond if the patient’s identity is not disclosed.
The police need to be informed quickly in order to respond to the risk to patients, staff, and the public.
They also need statistical information about the number of gunshot and knife injuries, and when and where they occur, to inform their own and their crime reduction partners’ operational and strategic priorities.
Q.11. A 39 week pregnant, 28 year old woman in labour is in the maternity unit in a second stage of labour. The cardiotocography (CTG) shows late decelerations in over 50% of contractions for more than 30 minutes which suggest foetal distress. You have explained the situation to her and made it clear that an emergency caesarean-section is required and have informed her of the risk. She considers the consent but after a long discussionwith her husband andweighing out the risk, she decides that she still wants a vaginal delivery. A mental capacity assessment is carried out and she is deemed to have capacity. What is the most appropriate course of action?
Correct Answer : D
As she is capable of making decisions, we should respect her decision. A fetus does not hold any human rights until born. Her husband cannot consent on her behalf unless he has a lasting power of attorney.
There are 5 key principles for the Mental Capacity Act :
A person must be assumed to have capacity unless it is established that he lacks capacity A person is not to be treated as unable to make a decision unless all practicable steps to help him to do so have been taken without success.
• A person is not to be treated as unable to make a decision merely because he makes an unwise decision.
• An act done, or decision made, under this Act for or on behalf of a person who lacks capacity must be done, or made, in his best interests.
• Before the act is done, or the decision is made, regard must be had to whether the purpose for which it is needed can be as effectively achieved in a way that is less restrictive of the patient’s rights and freedom of action. The one key component to highlight in this situation is that a person is not to be treated as unable to make a decision merely because he makes an unwise decision.
Q.12. A 34 year old man comes to Accidents and Emergency with a knife wound on his arm. He tells you that he obtained it during a fight in the local pub. An hour later, the police arrive asking staff members for details as there was a murder at the pub and they are investigating it. What is the most appropriate action?
Correct Answer : C
Doctors have a duty of confidentiality which they owe to their patients, however, doctors should also realize that they must protect the public. In this stem, it is unclear if providing information to the police would in any way protect the public. It clearly states that when the police arrive, doctors should not allow them to access the patient if this will delay or hamper the treatment or compromise the patient’s recovery.
The background history in this question does not show any evidence that the police speaking to this patient, will directly compromise his recovery or delay treatment. It is therefore appropriate to allow the police to access the patient however one needs to obtain the patient’s permission to do so.
If the patient’s treatment and condition allow them to speak to the police, you or another member of the healthcare team should ask the patient whether they are willing to do so.
Q.13. A 30 year old female is brought into the Emergency Department by her husband drifting in and out of consciousness following a road traffic accident. A FAST scan reveals a large collection of free fluid in the peritoneal cavity. She is in shock and requires immediate blood transfusion. Her husband objects saying they are devout Jehovah’s Witnesses and are against blood transfusion. Her condition is deteriorating. What is the most appropriate action?
Correct Answer : D
In this emergency situation, there are a few reasons why a doctor should be transfusing blood without consent. Firstly, the patient is not conscious enough to make an informed decision pertaining to blood transfusion. Her husband in this case is not permitted to make that decision for her as there is no indication in the case stem that there is an advance directive allowing her husband to do so.
In this case, there is no mention of a blood refusal card. Therefore, you would treat this patient just like any other unconscious patient who has entered the Emergency Department until she can have the capacity to make an informed decision.
Q.14. A 45 year old man is found to be HIV positive in a Genitourinary Medicine clinic. He is not sexually active. He does not want his GP informed about his diagnosis as he has witnessed a friend who suffered discrimination following accidental disclosure of his HIV status. What is the most appropriate action?
Correct Answer : D
The patient’s decision must be respected unless there is an overriding public interest in disclosing the information without his consent. An overriding public interest constitutes a serious communicable illness such as tuberculosis or other notifiable disease. An HIV-positive patient may not need to reveal his diagnosis to the GP.
He may obtain his anti-viral treatments from the clinic.
Q.15. A 44 year old man with learning difficulties presents to the Emergency Department with vomiting blood earlier in the day. On admission, hisbloods show: Haemoglobin 99 g/L He is clinically stable but continues to vomit and requires an endoscopy to investigate the bleeding. Unfortunately, he is unable to understand the procedure or make an informed decision. What is the most appropriate action?
Correct Answer : C
The endoscopic procedure is in this patient’s best interest. If a person becomes incapable of making responsible decisions due to a mental disability, the court would appoint a substitute decision maker in advance, called the Guardian. This Guardian with power of attorney has the authority to make legal, financial, and healthcare decisions should the patient become incapable of making decisions about his health.
Q.16. An 82 year old woman with a background history of long standing dementia comes to the hospital with symptoms of pneumonia. She rapidly deteriorates after a few days. The medical team has deemed that resuscitation would not be in the patient’s best interest as it would lead to poor quality of life. Tehre is no advanced notice or living will in her notes which states her decision for resuscitation. Her daughter tells you that she thinks she should be resuscitated. What is the most appropriate action in regards to a Do Not Resuscitate (DNR) order?
Correct Answer : B
The GMC has very clear guidelines on what to do in scenarios in which determining capacity is not straightforward. Above all, you must always seek to do what is in the patient’s best interests. This may seem counter-intuitive as you will not be resuscitating this patient despite the daughter’s wishes, however, this is the correct answer.
Q.17. A 15 year old girl is brought to the hospital by her parents with mild lower abdominal pain. An ultrasound scan reveals a large ovarian cyst. The decision for a laparoscopic ovarian cystectomy has been made but she refuses treatment. She understands the consequences of not having the surgery and is deemed competent. Her parents ask if they can override her decision and sign the consent form on her behalf. What is the most appropriate action?
Correct Answer : A
Remember to note these few points in the stem that would help you answer your question 1. The case above is not an emergency and you have time to seek legal advice 2. She is deemed competent even though she is below 16 years old.
Q.18. A 33 year old man is extremely thankful for your service and for assisting in his laparoscopic appendectomy. He offers you a valuable clock as a gift. What is the most appropriate action for you to take?
Correct Answer : B
It may be entirely reasonable to accept a small token of appreciation from a patient. Refusing it may cause embarrassment that could unnecessarily harm the relationship between a doctor and a patient. But gifts of money or items of high financial value raise contractual and ethical issues that doctors need to consider. The GMC states that doctors must not accept gifts from patients, or colleagues if it is an inducement, gift, or hospitality that may affect or be seen to affect the way that person is treated.
Gifts with considerable value may alter the dynamic between the doctor and patient such that the patient may receive or expect to receive preferential treatment. Explaining to the patient that receiving such an expensive gift would not be in keeping with the guidance set by the General Medical Council,
The first action should always be refusing the gift. If this fails and the patient insists on giving, then you can suggest other options like giving it to charity or the ward fund.
Q.19. A 31 year old Jehovah’s witness was in a car accident and brought into the Emergency Department. He has lost a massive amount of blood. In the Emergency Department, he is given intravenous fluids. He is in critical condition and blood products are needed but he refuses any form of blood products due to his religious beliefs. What is the most appropriate next step?
Correct Answer : B
Jehovah’s Witnesses and Blood Transfusion : You must respect a competent patient’s decision to refuse an investigation or treatment, even if you think their decision is wrong or irrational. You may advise the patient of your clinical opinion, but you must not put pressure on them to accept your advice. You must be careful that your words and actions do not imply judgment of the patient or their beliefs and values.
Q.20. You are the GP working in psychiatry. Your consultant is having a sexual relationship with a widowed patient that is currently being treated for depression. The lady’s condition is much improved and is awaiting discharge next week. What is the most appropriate action?
Correct Answer : B
As per ethical guidance, as a medical professional, one must not “pursue a sexual or improper emotional relationship with a patient or someone close to them”.
In this instance, you must report the incident to the appropriate seniors in your hospital as per hospital protocol to investigate the allegation. Police would only be involved if you suspect that the consultant has committed sexual assault or other criminal activity towards the patient.
Q.21. A 32 year old woman is brought in unconscious by the ambulance after receiving the news that she has terminal breast cancer eaerlier in the day. She is suspected of having taken an overdose of benzodiazepines. She was found by her boyfriend with an empty bottle of tablets beside her with a note declaring that she wishes to end her life. The note specifically says that she does not want any treatment. What is the most appropriate action?
Correct Answer : D
The question here is how binding is the note that states she is refusing treatment.
A suicide note in most situations is NOT a valid advance decision. Decisions refusing life-sustaining treatment must be in writing and include a clear and specific statement that applies even if the patient’s life is at risk. It is unlikely that the note fulfills all these criteria. The document must also be signed and witnessed which in this stem is not.
Q.22. A 37 year old female who was recently diagnosed with multiple sclerosis took 100 tricyclic antidepressant tablets. She is now refusing all treatment. What is the most appropriate action?
Correct Answer : D
The work’s guidance on capacity seeks to preserve patient autonomy above all else. Some important points include:
• You must assess a patient’s capacity to make a particular decision at the time it needs to be made. You must not assume that because a patient cannot decide on a particular occasion, they cannot make any decisions at all, or will not be able to make similar decisions in the future.
• If your assessment leaves you in doubt about the patient’s capacity to make a decision, you should seek advice from: Nursing staff or others involved in the patient’s care, or those close to the patient, who may be aware of the patient’s usual ability to make decisions and their particular communication needs o colleagues with relevant specialist experience, such as psychiatrists, neurologists, or speech and language therapists.
• If you are still unsure about the patient’s capacity to make a decision, you must seek legal advice to ask a court to determine capacity.
Q.23. An 85 year old woman with Alzheimer’s disease attends her GP surgery with her daughter for her annual medication review. During the consultation, the patient tells you that she is planning on changing her will. Her granddaughter is refusing as she says that her grandmother does not know what she is saying since she is suffering from dementia. What is the most appropriate action to take?
Correct Answer : C
Referring the patient for assessment of her capacity is the best possible choice out of the given choices. A safe answer for exam ethics questions is always discussing and exploring patient concerns. Allowing her to change her will would be the correct choice if she wasn’t diagnosed with Alzheimer’s disease.
Patient autonomy is the founding principle of medical practice and we should always endeavor to maintain patient autonomy at all times. Refusing her to change her will is the incorrect choice because, without an evaluation of whether she has the capacity or not, this would be a violation of the code of ethics.
There was no mention in the stem that her granddaughter is indeed her caretaker, and even if her granddaughter was her caretaker, important decisions that impact the patient cannot be undertaken by a caretaker without a power of attorney document (which also was not mentioned in the stem).
Q.24. 14 year old girl presents to the clinic asking for contraception. She is sexually active. When asked about her partner, she initially did not want to reveal him but eventually she tells you that her partner is a teacher in her school. She was previously given advice to inform her parents which she has refused to do so. She clearly states that she would not want anyone to know about her relationship with the teacher. What is the most appropriate action?
Correct Answer : C
This is a clear picture of the exploitation of a young child by a teacher. It is a crime to be involved with children under 16 years old. Your first concern should be the safety of the child. You must inform an appropriate person or authority promptly of any reasonable concern that this child is at risk of abuse. In regards to confidentiality, in this case, you should share information as this could be an abusive or seriously harmful sexual activity involving the young person. The teacher has a position of trust.
This position should not be abused and you would need to report this. The main confusion is the option of informing the safeguarding authority or informing the police. Informing the local safeguarding authority would be the first step and they would usually involve the police and family if necessary.
Q.25. A 33 year old pregnant woman is in labour at 40 weeks gestation. The emergency bell was rung by the midwife in the labour room as the cardiotocograph shows fetal bradycardia for the past 5 minutes that has not recovered. A quick decision to go for an emergency C-section is made however she is unable to consent as she speaks only Tigrinya and cannot understand English. Her partner is not in the hospital with her. Attempts have been made to contact the language line but at present they are unable to obtain a Tigrinya translator. What is the most appropriate action?
Correct Answer : B
It is clear that doctors must have consent before carrying out any treatment. However, in an acute emergency like this stem, the importance of saving the fetus’ life outweighs this. It would be considered wrong not to act on fetal bradycardia on the basis that the mother is unable to understand the language. Waiting for a translator may put the fetus’ life at risk. Fetal bradycardia is an acute emergency and obstetricians are trained to get the baby out by C-section within 60 seconds of skin incision.
No time should be wasted trying to seek legal advice or contacting the next of kin. Furthermore, the mother has not refused a C-section, she is merely unable to understand the language. In the absence of a clear expression of her wishes, you may act and provide appropriate treatment and prevent serious detriment to the baby.
Q.26. One of the nurses at the end of her shift, accidentally puts a copy of a list containing handover details of all the patients in the ward into handheld notes of a patient. The patient unknowingly takes the list home and calls the ward the following day informing you of the incident. What is the most appropriate action?
Correct Answer : D
The Information Governance Lead is in charge of the collection, storage, and management of information. As this is a breach of confidentiality, the Information Governance Lead needs to be aware of this so that they can take appropriate action.
Q.27. A 33 year old man is brought into the Emergency Department having been involved in a fight outside a local pub. During the fight, he was hit on the head by a metal chair. On examination, there is an open laceration on his head and he has sustained cuts and bruises on his arms and legs. He also smells of alcohol. He was initially uncooperative before losing consciousness. What is the most appropriate action?
Correct Answer : A
It is clear that doctors must have consent before carrying out any examination, investigation, or providing treatment. However, it is likely that the patient’s initial refusal was due to his condition rather than the fact that he was refusing consent for investigations and treatment. As the patient has sustained a head injury it is not practical to wait until he is sober before obtaining consent as his life could be at risk.
Neither is it appropriate to seek legal advice in an acute situation like this. Remember, he could be acutely deteriorating and early detection is vital. In the absence of a clear expression of his wishes, you may act and provide appropriate treatment and prevent a serious deterioration of his condition.
Q.28. A 15 year old girl is requesting for oral contraceptive pills (OCP) as she is sexually active. She refuses to tell her parents about her sexual activity and mentions that her partner is a 38 year old man. What is the most appropriate action?
Correct Answer : C
Under the Fraser guidelines, a competent young person may be advised on contraception. However, this man is much older than her. Thus, you would need to share information regarding this and breach confidentiality.
With a large age discrepancy between the teenager and her partner, the concern is the possibility of child abuse or exploitation If her sexual partner was of a similar age, for example, 15 years old, and she had the maturity to understand the advice and its implications, you should then advise them about safe sex, prescribe the COCP, and not breach confidentiality. The age discrepancy between the girl and her boyfriend, is seen as the boyfriend being able to be of greater influence and power over the girl.
As such, this would be considered a case of sexual abuse and needs to be reported to social services and the police. You would also provide contraception as requested by the girl as she is of mental capacity to make an informed decision over treatment, you would not be obligated to inform or gain consent from her parents for doing so.
However, you would advise the girl that it would be in her best interests to discuss with her parents.
Q.29. A 36 year old woman attends the gynaecology clinic with request for a laparoscopic sterilisation. She has two children and has completed her family. She does not want to use any other form of contraception and she understands the risk of a laparoscopic sterilisation. Her husband is unhappy with this decision and would like a further child. What is the most appropriate action?
Correct Answer : C
As the woman is undergoing the procedure and understands the risk of the procedure, her partner is not required to consent. The procedure would be performed on her body and her partner has no legal right to object to such a procedure. As this is a surgical intervention, a written consent preoperatively is required.
Q.30. You are working as a junior doctor in one of the medical wards. During your handover to one of your colleagues, you notice that he smells of alcohol. You suspect that he is under the influence of alcohol. You have not noticed this before. What is the most appropriate action?
Correct Answer : B
The first step when you suspect a colleague with an issue is to confront them to hear the whole story before judging. It may very well be that someone had spilled alcohol on him that day before he came to work. If his story was likely to be untrue or if you still had concerns about him with the possibility that he may be coming to work under the influence of alcohol, the next step would be to inform your senior doctors in charge. The risk of alcohol consumption prior to work coupled with the daily demands of doctors could lead to errors in their clinical judgment which could seriously jeopardise patient safety.
Your seniors would need to know and act on this. If you feel that you are unable to raise your concern with an appropriate person like your senior doctor for any reason, or if you do raise it but feel as if it has not been acted on appropriately, you can consider contacting the authorities directly.
Q.31. A 30 year old man is found to be HIV positive and is against condom use. After discussing with him, he is still refusing to accept the need to tell his wife of his diagnosis. What is the most appropriate action?
Correct Answer : D
There are partner notification services for those diagnosed with sexually transmitted infections (STI) and/or HIV. This service’s main aim is to inform unsuspecting partners of those infected of the possibility of being infected with STI/HIV and to seek medical care. Under normal circumstances, the patient affected is offered a choice: to inform their partner(s) themselves or to provide details of those partner(s) to the healthcare worker to contact them Batmanymously.
Usually, it is a mix of the above where the patient is given a time frame to inform their partner(s) and if not done by that set date, the healthcare worker will be the one to inform. In this case, the man refuses to tell his wife the diagnosis. As this is a communicable disease, the wife needs to be informed as she will need to be tested. Therefore, you would contact her via protocols of the partner notification program.
Q.32. You suspect that your colleague has been under the influence of illicit drugs however he denies the allegation after you have confronted him. You are still not convinced that he is telling the truth. What is the most appropriate action?
Correct Answer : B
You may think that this is a difficult ethics question but in reality, it is not. You suspect that your colleague is taking illicit drugs and have confronted him about it. There is no evidence to the contrary but you still suspect him of abusing illicit drugs. The guidelines are very clear on this issue.
Doctors should always remember their duty to raise concerns when they believe that a colleague may not be fit to practice or if they otherwise may pose a risk of serious harm to patients.
If you have a strong suspicion about your colleague, then you should not take him at his word.
The next step would be to discuss the situation with your senior. You should not attempt to gather more evidence by asking the nurses or informing your other colleagues about the situation. This would be unprofessional.
Q.33. You are a junior doctor working. You suspect one of your colleagues to be under the influence of recreational drugs. What is the most appropriate action to be taken?
Correct Answer : D
This is the best choice out of the given choices. In EXAM, ethics questions, it is always safe to discuss and explore concerns. The ethical guidance outlines state that:
• You should understand the difference between a personal grievance, which is a complaint about your employment situation, and a concern about a risk, malpractice, or wrongdoing that affects others. This is particularly important if patients or members of the public are at risk of harm
• You should be aware that poorly performing colleagues may have health problems and respond constructively where this is the case.
• You should encourage such colleagues to seek and follow professional advice and offer them appropriate help and support.
• You must not unfairly discriminate against colleagues because of an issue related to their health or a disability.
• You should, as far as possible, support colleagues who are experiencing performance problems. But, in all cases, you should remember your duty to raise concerns where you believe a colleague may not be fit to practice or may otherwise pose a risk of serious harm to patients.
Q.34. A 14 year old girl presents to the clinic requesting oral contraceptives. She is sexually active with her 15 year old partner. What is the most appropriate action?
Correct Answer : D
In this case, both the girl and her boyfriend are roughly the same age. There is no need to inform the police about their sexual activity as they are of similar age. If the boyfriend was much older, and thereby had a greater disparity in age to the girl, then informing the police and/or social services would be considered.
In this case, you are permitted to provide contraception to the girl without parental knowledge or consent as it is most likely in her best interests to do. Encouraging her to inform her parents is another point that a good practitioner would make.
Q.35. A 34 year old man with multiple sclerosis has taken an overdose of more than 50 tablets of paracetamol with the intent to end his life. He has been brought to the emergency department by his wife. He is refusing all interventions. What is the most appropriate action?
Correct Answer : C
The patient has multiple sclerosis. The first thing to do if he is refusing treatment is to evaluate his mental capacity to do so.
Q.36. A 31 year old woman comes to Accidents and Emergency for a worsening cough and rigors. During the consultation, her husband translates for her as she speaks no English at all. On auscultating her chest, you notice multiple old and new bruises on her chest. Her husband says that they were from a fall a week ago. What is the most appropriate immediate action?
Correct Answer : D
Domestic violence needs to be taken seriously. As doctors, we need to be vigilant and look out for cases like this. Admission overnight allows the woman to be alone away from her partner and an independent translator such as the “language line” can be used.
The “language line” is a 24-hour interpreting service that allows medical staff to communicate in almost all languages to their patients. You would need to call the hospital operator to put you through and the translator on the phone would help translate your conversations. One should find out the whole story before contacting the police or the safeguarding team.
Q.37. A 24 year old girl presents to the Emergency Department with her partner, mother and sister. The patient reports a one day history of lower abdominal pain. She has been amenorrhoeic for 10 weeks. A pregnancy test is positive. A speculum examination reveals an opened cervical os with active heavy bleeding. She was resuscitated with normal saline and given tranexamic acid however she remained hypotensive. It is clear that she needs to taken to theatre for evactuation of retained products of conception however she is semi-conscious, confused and unable to speak. What is the most appropriate action?
Correct Answer : D
In urgent scenarios like this where a patient is semi-conscious and unable to provide consent, we should take the patient to the theatre. If it is not possible to find out the patient’s wishes, treat them without their consent if it is to save their life.
Document that the patient is unable to consent because she is unable to comprehend the current scenario. After the procedure is performed and she gains consciousness again, she should be formally debriefed. Her family or her partner cannot consent on her behalf. Think about it, if we somehow lived in a world where they were allowed to consent on her behalf and they decided to say “no”, what would we do then?
Q.38. A 34 year old woman who is fully dilated in the labour and has an abnormal CTG. A decision for an emergency caesarean section is made however she refuses to sign the consent form as she has a fear of surgical intervention. She understands the risk for her unborn child and it is clear that she has a mental capacity to make a decision. What is the most appropriate action?
Correct Answer : A
This woman has all the right to refuse consent as long as she has the mental capacity to do so. An unborn child has no rights and so we would not be able to proceed with the caesarean section for the benefit of the baby. Her partner is unable to consent on her behalf.
Q.39. A 33 year old woman who is 39 weeks pregnant is in the labour ward with her husband. She is in early labour. The midwife presses the emergency alarm as she notices the umbilical cord prolapse out of the uterus before the presenting part. The senior obstetrician is informed and a decision for an emergency caesarean section is made and she is rushed into the theatre. Unfortunately, she is unable to sign the consent form as she is illiterate. What is the most appropriate course of action?
Correct Answer : C
In an ideal world, written consent is the most appropriate. However, if this is not possible because she is illiterate or because of the extreme urgency such as in this situation, verbal consent is appropriate. It is good practice to document in the patient’s notes the reason for verbal consent rather than written consent.
The gold standard management of umbilical cord prolapse is immediate delivery by the quickest and safest route possible which is usually caesarean section, especially if the woman is in early labor.
Her husband cannot consent on her behalf unless she cannot do so. Legal advice is not appropriate in an emergency nor is obtaining advice from the consultant where the senior obstetrician has declared this as an emergency cesarean section for cord prolapse.
Q.40. A 15 year old boy is brought to the hospital by his parents complaining of lower abdominal pains for the past three days. A clinical suspicion of appendicitis and the decision to admit was made. The young boy refuses to be admitted as he has plans to go out with his friends tonight. He is unable to understand the serious nature of an untreated appendicitis. His parents would like to overrule his wishes and to admit him. What is the most appropriate course of action?
Correct Answer : D
This young boy is under 16. He lacks maturity and is unable to understand the seriousness of his condition. You therefore can rely on parental consent.
Q.41. A 42 year old male patient has just been rushed into Accident & Emergency following a road traffic accident. He has broken his right clavicle and suffered some cuts and bruises, but is conscious, alert and oriented to place, person and time. His wife arrives at the hospital and comes to you at the waiting area, frantic, enquiring about the condition of her husband. What is the most appropriate action?
Correct Answer : B
The guidelines on confidentiality are clear. No information should be disclosed to third parties unless the patient has given explicit permission to the healthcare provider to do so. This includes giving a wife information about her husband’s condition without first consulting with her husband. Since this patient is alert, the most appropriate action would be to ask the patient’s permission to disclose information (including how much information) to his wife.
Understandably, relatives of patients will want to know about their loved one’s condition but it is important to keep the principles of confidentiality in mind when speaking to do this.
What about medical emergency cases where the patient is unconscious? In a scenario where the patient cannot be informed about the disclosure of his or her personal information (for example, a patient was hit by a car and is currently unconscious with his relatives waiting just outside the resuscitation area), you may still inform his relatives unless you have a reason to believe that the patient would object this.
Q.42. You are treating a general surgeon for Hepatitis B. You ask him to inform the hospital where he is currently working and he refuses. What is the most appropriate action?
Correct Answer : C
Under guidelines, it is the duty of a healthcare worker to report whether he or she has a serious communicable disease. If they do not, then you as their doctor must report it to the relevant health authorities as they can put the patients they are treating at risk.
Q.43. A healthy 19 year old female, gestational age of 27 weeks, presents to Obstetrics and Gynaecology clinic requesting a termination. She says her boyfriend has just left her and that she would not want the baby to remind her of him. The boyfriend, who is currently in jail, has reportedly been assaulting her during their stormy 2 year relationship. What is the most appropriate action?
Correct Answer : B
Under law, the Abortion Act states that termination of pregnancy is legal up to 24 weeks of gestation. Termination over 24 weeks can only be done under special circumstances (i.e. where the continuation of pregnancy would jeopardize the well-being of the mother and/or the birth of the child would result in his/her being seriously handicapped). In this case, the patient is healthy and there is no indication that her current pregnancy is jeopardizing her life nor is the fetus having any serious abnormalities.
As such, termination of pregnancy needs to be refused.
Q.44. A 15 year old girl comes in with her 15 year old boyfriend requesting for oral contraceptive pills (OCP). They are sexually active. They could not be persuaded to inform their parents about their sexual relationship or that contraceptive advice was being sought. What is the most appropriate action?
Correct Answer : A
This question tests your knowledge of the Fraser Criteria. The guidance states that the duty of confidentiality is the same for children and young people as it is for adults. Confidentiality may only be breached to protect the adolescent or others from serious harm usually in issues where child abuse and child protection are involved.
The guidance also states that ‘any competent young person, regardless of age, can independently seek medical advice and give valid consent to treatment’. Note that the ability to consent changes if you are under 13.
By law children under 13 are considered unable to consent.
Q.45. A 34 year old Jehovah’s Witness has a severe postpartum haemorrhage after a normal vaginal delivery. Her estimated blood loss is 2 litres and she is still having ongoing bleeding. She has an advance directive form stating she is not to receive blood products even if it is to save her life. She has a pulse rate of 90 beats/minute and a blood pressure of 105/60 mmHg. She is semi-conscious and is not speaking. Her husband is extremely distressed and would like something done. What is the most appropriate action?
Correct Answer : B
We should respect her decision and since she has an advance directive, we should accept that and not transfuse blood products. Nearly all Jehovah’s Witnesses refuse transfusions of whole blood (including preoperative autologous donation) and the primary blood components – red cells, platelets, white cells, and unfractionated plasma. Jehovah’s Witnesses consider plasma (including FFP) to be a primary component of blood which they do not accept.
Although she has signed an advance directive stating she is not to receive blood products, this does not mean we withdraw care. Administering intravenous fluids can help restore perfusion and prevent hypovolemic shock.
Q.46. A man with dementia has an ulcerative lesion on his forehead. He wants it removed so it can help improve his memory. His wife says he is not fit to give consent. What is the most appropriate action?
Correct Answer : C
The question is very straight forward. He is in no position to give consent so a psychiatrist should review his mental capacity.
Q.47. A 48 year old man recently had a transient ischaemic attack. What advice on driving would you give him?
Correct Answer : D
You must stop driving for at least 1 month after a transient ischaemic attack (TIA).
Q.48. A 15 year old girl attends the emergency department with mild vaginal bleeding. A pregnancy test is positive. She does not want her parents to know about her pregnancy and she refuses to disclose any information regarding her sexual partner to the healthcare professionals. What is the most appropriate management?
Correct Answer : A
Confidentiality under 16 : A young person under 16 has the right to advise and treatment without parental knowledge and consent provided that she is competent and understands all aspects of advice and its implications, and cannot be persuaded to tell her parents or to allow you to tell them. There are only certain circumstances in which you may disclose information that a young person does not agree to disclose.
These are :
• When there is an overriding public interest in the disclosure
• When you judge that the disclosure is in the best interests of the young person who does not have the maturity or understanding to make a decision about disclosure
• When disclosure is required by law Note that this only applies to children 13 and older.
Sexual activity with a child under 13 is a criminal offense and should always result in a child protection referral. You should usually share information about sexual activity involving children under 13, who are considered in law to be unable to consent.
Q.49. A 44 year old man has been brought in with an overdose of paracetamol. His blood reports show a paracetamol level above the cutoff to start N-acetylcystein infusion. However, he is refusing treatment as he is adamant that he wants to die. What is the most appropriate course of action?
Correct Answer : C
Please see Q-22
Q.50. You are a junior doctor eating with a few friends in a restaurant. On leaving the restaurant, you notice an empty table with patient’s medical notes. The table has been unattended for a while. What is the most appropriate action?
Correct Answer : D
Informing the GP practice would be the most practical option. The GP surgery would have all the records of the patient and they would be able to advise you what you need to do with the notes. It may be possible that the GP surgery would ask you to bring the notes into the practice and they will keep it securely or scan a copy to the hospital if needed.
The second most appropriate option is to bring the notes to the hospital. This is inconvenient as great efforts and time would be needed to find the department that this patient attended.
Q.51. A mother of a 14 year old girl attends your clinic to ask for advice on which type of contraception would be best suited for her 14 year old daughter whom she says suffers from dysmenorrhoea. She would like to obtain a prescription for her daughter as her daughter was not able to make the appointment due to school activities. What is the most appropriate next step to take?
Correct Answer : C
This ethics question is fairly straight forward. The next best step would be to invite the mother to attend the clinic with her daughter. It is always best to have parents involved in the care of young people, provided that the young person in question has the capacity and has provided the necessary consent.
At that consult with both of them present, you may wish to assess the daughter’s capacity and gain her consent for her mother to be involved in her care.
Q.52. A junior doctor had a needlestick injury from the needle used to draw blood from an unconscious patient in the intensive care unit. The unconscious patient is deemed to be a low risk of having blood-borne infections although he has not been tested for any blood-borne infections in the past. The junior doctor is worried about a blood-borne infection. What is the most appropriate action?
Correct Answer : C
As a health professional, receiving a needlestick injury from an unconscious patient where the blood-borne infection status is unknown can be stressful. The guidance points out that an unconscious patient should not be tested for blood-borne infections if it is not in the patient’s medical interest.
However, patients can be tested prior to their consent for blood-borne infections when it is in their immediate medical interest.
Taking blood in this scenario to test for serious communicable diseases is not in the patient’s best interest but the junior doctor’s best interest hence waiting until the patient is conscious to obtain consent is the correct action.
There are certain exceptions where one may use an existing blood sample that has been taken for other purposes to test for conditions such as HIV.
One example is if there is a good reason to believe that the donor may have HIV. These circumstances are complex and should be discussed with a senior colleague first as this decision may be challenged in court.
Q.53. A 15 year old girl presents with her boyfriend to the Paediatric Accidents & Emergency with lower abdominal pain. On routine history taking, she mentions that she is sexually active with her boyfriend who is a 23 year old college student. Her parents have no knowledge of her sexual activity. Her boyfriend is aware of her actual age. After a full examination, she is deemed medically fit for discharge. What is the most appropriate step to perform prior to discharging the patient?
Correct Answer : A
Since the girl is above 13 years of age but below 16 years of age, this 23-year-old male is guilty of “unlawful sexual intercourse”. There are two defences in this offense which are if he believes that she is his wife or if he reasonably believes that she is over 16. Both of which are not seen here.
Even if they were present here, the answer would remain to inform the safeguarding authority.
This man is much older than the patient, therefore, breaking confidentiality is appropriate to ensure this child is not exploited. The main confusion in this question is the option of informing the safeguarding authority or informing the police.
The safeguarding authority would be more appropriate in this setting as there is always a safeguarding team located in the hospital who can approach the patient and take a full detailed history.
This is often the first step. The safeguarding team would involve the police and family if necessary after reviewing the patient.
Q.54. You are taking a hard-earned break in the doctor’s lounge during a busy shift when you notice your colleague, browsing what appears to be photographs of X-rays on his smartphone. When you confront him, he tells you that he frequently takes photographs of patients’ X-rays. He tells you that it is for his personal study since he is aspiring to be a consultant radiologist one day. He assures you that he will not share or send the photographs to anyone. What is the most appropriate action that you should take in this situation?
Correct Answer : B
This question is about commitment to professionalism. It also tests your understanding of the principles of confidentiality. The most appropriate answer amongst the options is to inform the colleague in question that what he is doing is wrong and advise him to delete or dispose of confidential patient information.
This would mean taking X-ray photographs without the patient’s name, hospital number, or date of birth in the pictures.
For cases where it is not possible to Batmanymize the information (e.g. taking photographs of patients’ faces for skin diseases), then consent from the patient should be obtained.
It is not appropriate to inform his educational or clinical supervisor yourself.
The only time that this would be appropriate would be if he continues this behaviour despite multiple warnings.
It is appropriate to advise him to talk to his clinical supervisor about the situation but a better answer would be to talk to him first and reiterate the principles of confidentiality.
Q.55. An elderly man who used to work in the shipyard industry presented with cough and SOB (shortness of breath) few weeks to months. He was given salbutamol nebulization and antibiotics and admitted to the ward. He died 3days later. CT thorax: patchy infiltrates, pleural thickening and pleural effusion. Why is this a coroner’s case?
Correct Answer : C
Death could be due to occupational illness.
Q.56. A man with dementia has an ulcerative lesion on his forehead. He wants it removed so ‘it can help improve his memory’. Wife says he is not fit to give consent. What will you do?
Correct Answer : D
Refer to psychiatrist to assess the mental capacity to give consent.
Q.57. A 78-year-old patient is diagnosed with metastatic lung cancer; there is no cure for his condition. His son tells the physician that in the case of a diagnosis of cancer, the physician must not tell his father. He wishes that his father does not suffer any psychological distress caused by the knowledge of a terminal diagnosis. Which one of the following ethical principles supports the son’s request?
Correct Answer : D
Nonmaleficence means non-harming or inflicting the least harm possible to reach a beneficial outcome.
Q.58. A 45-year-old woman underwent abdominal surgery. After the procedure, she developed severe abdominal pain and fever. Imaging revealed a retained surgical sponge. She was urgently taken back to the operating room for removal. As the assisting doctor, what is the most appropriate immediate next step?
Correct Answer : B
Correct Answer:
B. Inform the patient and offer an apology
Explanation:
In the event of a medical error, particularly one involving patient harm, ethical and professional guidelines mandate prompt disclosure to the patient. This includes:
-
Informing the patient of what occurred.
-
Offering a clear and honest apology.
-
Explaining the steps being taken to correct the issue and prevent recurrence.
Most major ethical bodies, including the World Medical Association and American Medical Association, emphasize transparency and accountability as central to patient care. Waiting for a consultant or hiding behind protocols is not ethically or legally acceptable.
Rule-out of Other Options:
-
A. Inform the consultant:
While informing the consultant is necessary, it is not the most appropriate immediate action. Patient disclosure must not be delayed. -
C. Inform the hospital ethics committee:
Ethics committees can assist in managing complex ethical dilemmas, but they are not the first point of action in a clear-cut case like this. -
D. Document it in the notes and wait for the consultant to speak to the patient:
This delays necessary disclosure and accountability. It also violates ethical standards for timely patient communication.
Clinical Tip:
Ethics Principle: "Tell the truth, take responsibility, make it right."
Medical errors must be disclosed directly and compassionately to the patient. Doing so builds trust and upholds professional integrity. Apologies are not only ethical—they’re often legally protective when done correctly.
Q.59. A 34-year-old woman presents to the clinic with persistent worry, muscle tension, and difficulty sleeping for the past 6 months. She reports feeling “on edge” most of the day. She has no history of substance use or depression. She was recently diagnosed with generalized anxiety disorder (GAD). Cognitive behavioral therapy (CBT) has been recommended but she seeks pharmacologic treatment to help with immediate relief of insomnia and anxiety symptoms. Which of the following medications is most appropriate?
Correct Answer : A
Correct Answer:
A. Alprazolam
Explanation:
Alprazolam is a short-acting benzodiazepine, often used for short-term relief of acute anxiety symptoms and sleep disturbances in patients with GAD. While SSRIs or SNRIs (e.g., sertraline, venlafaxine) and CBT are considered first-line treatments, benzodiazepines like alprazolam can be used short-term to relieve severe anxiety or insomnia while awaiting response to other therapies.
Important: Benzodiazepines should be used cautiously and short-term only, due to risks of dependence, tolerance, and withdrawal.
Rule-out of Other Options:
-
B. Bupropion:
An atypical antidepressant that is activating, not typically used for anxiety. It can worsen anxiety and insomnia due to its stimulating properties. -
C. Haloperidol:
A typical antipsychotic, used for psychotic disorders and delirium, not for anxiety. It has no role in treating GAD and may have serious side effects (e.g., extrapyramidal symptoms). -
D. Lithium:
A mood stabilizer used in bipolar disorder, not indicated for GAD.
Clinical Tip:
Mnemonic to recall short-term GAD relief:
“Benzo for the bridge” – Benzodiazepines (e.g., alprazolam) bridge the gap until SSRIs or therapy take effect.
However, CBT remains the gold standard for long-term management. Always assess risk of dependence when prescribing benzodiazepines.
Q.60. An 80-year-old man is diagnosed with metastatic pancreatic cancer. He is alert and mentally competent. His son approaches the physician privately and requests that his father not be informed of the diagnosis, fearing it will emotionally devastate him. What is the most appropriate next step?
Correct Answer : A
Correct Answer:
A. Disclose the diagnosis to the patient
Explanation:
Patient autonomy is a foundational ethical principle in medicine. A competent adult has the right to know their medical diagnosis, prognosis, and treatment options. While family concerns may be well-meaning, withholding a diagnosis from a competent patient is a violation of informed consent and ethical standards.
The physician should:
-
Respect the patient’s right to know.
-
Speak directly with the patient to determine how much he wants to know.
-
Reassure the family that the discussion will be handled sensitively.
If the patient expresses a preference not to know, then withholding information is ethically acceptable—but only if this is the patient’s own wish.
Rule-out of Other Options:
-
B. Consult the legal committee:
This is unnecessary. No legal conflict exists. The patient is competent, and the law supports disclosure. -
C. Respect the family's wishes and withhold the diagnosis:
Ethically and legally wrong. The family cannot override the patient’s right to know. -
D. Refer the case to the hospital ethics committee:
Ethics consults are useful in complex or uncertain cases. Here, the ethical obligation is clear: inform the patient.
Clinical Tip:
Autonomy > Family Wishes
If the patient is mentally competent, the healthcare provider must respect their autonomy. Always assess the patient’s desire for information directly—don’t assume based on family input.
Q.61. A general practitioner evaluates a patient with a suspected viral upper respiratory infection. At the end of the consultation, the doctor advises the patient to return if they develop high fever, shortness of breath, or persistent symptoms beyond a few days. What is this communication strategy called?
Correct Answer : A
Correct Answer:
A. Safety netting
Explanation:
Safety netting is a communication strategy used to inform patients about:
-
What symptoms to watch for
-
When to seek further medical help
-
Expected course of the illness
It is crucial in primary care and emergency settings where many conditions are managed symptomatically or when serious illness cannot yet be ruled out. Safety netting empowers the patient, improves outcomes, and reduces medicolegal risk.
Rule-out of Other Options:
-
B. Informed consent:
This refers to explaining risks, benefits, and alternatives before a procedure or treatment—not general follow-up advice. -
C. Shared decision-making:
Involves clinician and patient making joint medical decisions based on clinical evidence and patient values—distinct from post-visit guidance. -
D. Health promotion:
Focuses on preventive measures like diet, exercise, smoking cessation—not the management of uncertain illness progression.
Clinical Tip:
Mnemonic: “3 C’s of Safety Netting”
-
Course: What to expect in recovery
-
Complications: What red flags to look out for
-
Contact: When and how to seek help
Use this in every uncertain or evolving diagnosis—especially in outpatient or emergency settings.
Q.62. A 67-year-old man with a history of cognitive decline frequently asks strangers inappropriate personal questions in public. He repeats the same behavior despite being told it is socially unacceptable and seems unaware of the inappropriateness of his actions. What is the most likely underlying thought or behavioral pattern?
Correct Answer : C
Correct Answer:
C. Perseveration
Explanation:
Perseveration is the persistent repetition of a word, idea, or behavior, even when it is no longer appropriate or relevant. It is commonly seen in:
-
Frontal lobe dysfunction
-
Dementia, especially frontotemporal dementia
-
Certain psychiatric conditions
In this case, the patient continues to ask inappropriate questions and fails to adapt his behavior despite correction, indicating behavioral perseveration, not just a speech abnormality.
Rule-out of Other Options:
-
A. Flight of ideas:
Seen in mania, characterized by rapidly shifting thoughts with superficial connections—but not repetitive or socially inappropriate behavior. -
B. Loosening of associations:
Seen in schizophrenia, where the patient’s thoughts are disorganized and illogically connected, but not necessarily repetitive or perseverative. -
D. Echolalia:
Refers to repetition of another person’s spoken words, often seen in autism or catatonia—not relevant in this case.
Clinical Tip:
Perseveration = Repetition without adaptation
It reflects frontal lobe dysfunction, often in dementia or neurologic injury. Look for rigid, repetitive behavior that continues despite correction or context.
Q.63. When screening for a rare disease in the general population using a test with high sensitivity, which type of result is expected to be the most common among the positive results?
Correct Answer : A
Correct Answer:
A. False positive
Explanation:
In the context of screening a rare disease (i.e., very low prevalence), even a highly sensitive and specific test will yield more false positives than true positives. This is due to Bayes’ Theorem, which highlights the effect of disease prevalence on positive predictive value (PPV).
-
In a rare disease, very few people actually have the disease.
-
Therefore, among the many who test positive, most will not actually have the disease.
-
This causes PPV to be low, and false positives to dominate among positive results.
Rule-out of Other Options:
-
B. True positive:
True positives are few when the disease is rare, even if the test is good. -
C. False negative:
Unlikely if the test is highly sensitive. False negatives are minimized. -
D. True negative:
While true negatives are numerous in rare diseases, the question asks about positive results, so this is not applicable.
Clinical Tip:
Rare Disease Rule:
"In a low-prevalence population, most positive results will be false positives—even with a good test."
That's why mass screening for rare diseases must be approached cautiously and often avoided unless the condition is highly impactful and treatable.
Q.64. A medical student enters a ward and begins to discuss a patient’s condition with classmates while the patient is present but undressed. Which principle is most clearly violated?
Correct Answer : B
Correct Answer:
B. Dignity
Explanation:
The patient is exposed and not being treated with respect and modesty, which violates dignity.
-
If personal information were disclosed without consent, that would violate privacy.
-
Autonomy and informed consent relate more to decision-making.
Clinical Tip:
Dignity is about how you treat the patient; privacy is about what you expose or share.
Q.65. During a physical examination, a doctor forgets to close the curtains around the patient’s bed, exposing the patient to others in the room. Which ethical principle is most violated?
Correct Answer : A
Correct Answer:
A. Privacy
Explanation:
-
Privacy refers to protecting the patient’s physical space and confidentiality, such as closing curtains or doors during examination.
-
Dignity refers to respecting the patient’s modesty, like covering with clothes or gowns during a procedure.
In this scenario, not closing the curtains compromises the patient’s privacy by exposing them to others.
Rule out of other option:
-
B. Dignity:
Would be violated if the patient was not properly covered or treated with disrespectful behavior.
Clinical Tip:
Mnemonic:
-
Curtains = Privacy
-
Clothes = Dignity
Always close curtains or doors before examining and cover patients appropriately to uphold both privacy and dignity.
Q.66. A child is brought to the clinic with multiple bruises in various stages of healing. What is the most appropriate immediate action for the doctor?
Correct Answer : C
Correct Answer:
C. Inform child protection services
Explanation:
-
Multiple bruises in different stages of healing may indicate child abuse or non-accidental injury.
-
The doctor’s legal and ethical duty is to report suspicions to child protection services promptly.
-
Direct questioning may put the child or family at risk and could compromise investigations.
-
Referral is important but secondary to immediate safeguarding through reporting.
Rule out other options:
-
A. Question parents about bruises: May delay reporting and could cause harm or tampering.
-
B. Refer the child: Specialist referral is important but only after reporting to authorities.
Clinical Tip:
“When in doubt, report!”
Always prioritize the child’s safety by reporting suspected abuse to the appropriate authorities without delay.
Q.67. An elderly patient with advanced metastatic cancer is scheduled for surgery. The anaesthetist believes the operation is not advisable due to high risk, but the surgeon insists on proceeding. What is the most appropriate next step?
Correct Answer : A
Correct Answer:
A. Take a high-risk informed consent from the patient
Explanation:
-
When there is a disagreement but the surgeon decides to proceed, the patient must be fully informed about the high risks and give informed consent.
-
This protects patient autonomy and medico-legal safety.
-
Involving another anaesthetist or referring the patient can be options but do not replace the need for thorough informed consent.
-
Ethics committee involvement is usually for unresolved or complex ethical dilemmas, but in this case, patient consent is the immediate priority.
Rule out other options:
-
B. Surgeon operating with another anaesthetist: Not standard; anaesthetist must clear patient safety.
-
C. Inform ethics committee: May be considered if conflict persists but not the first step.
-
D. Refer patient: Not mandatory unless patient or surgeon requests.
Clinical Tip:
High-risk consent must clearly explain:
-
Nature of procedure
-
Risks and possible outcomes
-
Alternatives including no surgery
This is essential especially when clinical team members disagree.
Q.68. Early identification of a disease through screening tests primarily represents which type of prevention?
Correct Answer : B
Correct Answer:
B. Secondary prevention
Explanation:
-
Secondary prevention aims to detect and treat a disease early before symptoms develop to reduce severity and complications.
-
Screening tests (e.g., mammography, Pap smear) are typical examples.
-
Primary prevention involves preventing disease onset (e.g., vaccination).
-
Tertiary prevention focuses on managing established disease to prevent complications (e.g., rehabilitation).
-
Quaternary prevention aims to avoid unnecessary medical interventions.
Rule out other options:
-
A. Primary prevention: Prevents disease occurrence, not detection.
-
C. Tertiary prevention: Post-diagnosis care.
-
D. Quaternary prevention: Prevents harm from overmedicalization.
Clinical Tip:
Remember the prevention ladder:
Primary = prevent occurrence,
Secondary = early detection,
Tertiary = reduce impact.
Q.69. A patient’s wife refuses a recommended medical treatment for her husband. Who has the legal and ethical right to make the decision regarding the patient’s treatment?
Correct Answer : C
Correct Answer:
C. The patient
Explanation:
-
The patient has the primary right to make decisions about their own medical treatment if they are competent.
-
Family members cannot override a competent patient's autonomous decision.
-
Physicians provide information and recommendations but do not decide for the patient.
-
Ethics committees may provide guidance but do not replace patient autonomy.
Rule out other options:
-
A. The wife: Has no legal right to refuse treatment on behalf of a competent adult patient.
-
B. The physician: Advises and treats but does not have decision-making authority.
-
D. Ethics committee: Advisory role only.
Clinical Tip:
Always respect patient autonomy unless the patient lacks decision-making capacity.
Q.70. An elderly patient is diagnosed with atrophic vaginitis but expresses doubt and requests a second opinion. What is the appropriate response?
Correct Answer : B
Correct Answer:
B. Write a detailed report and refer her for another consultation
Explanation:
-
Respecting patient autonomy includes supporting their right to seek a second opinion.
-
Providing a clear report and referral ensures continuity of care and builds trust.
-
Denying or dismissing the patient’s request can damage the doctor-patient relationship.
Rule out other options:
-
A. Tell her second opinion unnecessary: Dismissive and undermines autonomy.
-
C. Refuse: Ethically inappropriate; patients have the right to choose their care.
Clinical Tip:
Encouraging second opinions improves patient satisfaction and may uncover new perspectives.
Q.71. A consultant instructs a resident to prescribe piperacillin-tazobactam for a patient. The pharmacist alerts that the patient has a penicillin allergy. What should the resident do?
Correct Answer : A
Correct Answer:
A. Refuse to write the prescription gently
Explanation:
-
The resident has a duty to patient safety and must avoid prescribing medications with known allergies.
-
A polite refusal is appropriate; the resident should explain concerns and seek clarification or alternative treatment.
-
Writing the prescription despite allergy risks harming the patient and is unethical.
-
Informing supervisors after prescribing does not prevent harm beforehand.
-
Refusing without explanation may cause conflict; communication is key.
Rule out other options:
-
B. Write then inform: Unsafe; patient risk not addressed immediately.
-
C. Write and ask consultant to sign: Shifts responsibility but does not resolve allergy risk.
-
D. Refuse without explanation: Poor communication and teamwork.
Clinical Tip:
Always prioritize patient safety and communicate concerns clearly and respectfully with the team.
Q.72. An elderly patient with advanced cancer is scheduled for surgery. The anaesthetist believes surgery is not advisable due to high risk, but the surgeon insists on proceeding. What is the most appropriate next step?
Correct Answer : A
Correct Answer:
A. Take a high-risk informed consent from the patient
Explanation:
-
When there is a disagreement but surgery is planned, fully informing the patient about the increased risks and obtaining informed consent is crucial.
-
This respects patient autonomy and provides legal protection.
-
Changing anaesthetists or referral may be options, but consent must always be obtained first.
-
Ethics committee involvement may be necessary if conflict persists but is not the immediate step.
Rule out other options:
-
B. Surgeon operating with another anaesthetist: Not standard practice; anaesthetist clearance is vital.
-
C. Inform ethics committee: Useful for unresolved disputes but not immediate.
-
D. Refer patient: Possible but consent remains priority.
Clinical Tip:
High-risk consent must clearly cover:
Nature of procedure
Risks, including mortality
Alternatives, including no surgery
Q.73. A 15-year-old patient is scheduled for surgery. From whom should consent be obtained?
Correct Answer : C
Correct Answer:
C. Both – Consent from parents and assent from the child
Explanation:
-
Legally, for minors (usually under 18), parents or legal guardians provide formal consent.
-
However, ethical practice involves obtaining the child’s assent (agreement) when they are mature enough to understand.
-
This respects the developing autonomy of adolescents.
-
Social workers may be involved in special situations but are not primary consent providers.
Rule out other options:
-
A. Parents only: Legally correct but does not respect child’s autonomy.
-
B. Child only: Usually not legally valid alone.
-
D. Social worker: Not a standard consent provider.
Clinical Tip:
Always involve the adolescent in discussions about their care to promote understanding and cooperation, even if legal consent is from parents.
Q.74. A patient diagnosed with ovarian torsion refuses to sign the consent form for surgery, but her husband insists on signing on her behalf. Who legally has the authority to sign the consent?
Correct Answer : A
Correct Answer:
A. Patient
Explanation:
-
A competent adult patient has the absolute right to refuse or consent to medical treatment.
-
Spouse or family members cannot override a competent patient’s decision.
-
Doctors must respect patient autonomy and cannot sign consent on behalf of patients.
-
Only if the patient lacks capacity can a legally authorized representative provide consent.
Rule out other options:
-
B. Husband: No legal authority if patient is competent.
-
C. Family member: Same as above.
-
D. Doctor: Cannot consent on behalf of a patient.
Clinical Tip:
Always confirm patient capacity and respect their autonomy regardless of family pressures.
Q.75. A married female diagnosed with ovarian torsion refuses to sign the consent form for surgery. From whom should you take consent?
Correct Answer : A
Correct Answer:
A. Patient
Explanation:
-
The competent adult patient has the primary right to consent or refuse medical treatment.
-
Marital status does not transfer consent authority to the husband.
-
Ethics committee involvement may be needed for complex cases but is not the primary source of consent.
-
Respect for patient autonomy is paramount.
Rule out other options:
-
B. Ethics committee: Advisory role, not primary consent provider.
-
C. Husband: No legal right to consent if patient is competent.
Clinical Tip:
Always respect patient autonomy, even when family members disagree.
Q.76. A patient’s husband is diagnosed as HIV positive. What is the most appropriate next step?
Correct Answer : A
Correct Answer:
A. Inform the husband
Explanation:
-
The first step is to inform and counsel the HIV-positive individual about the diagnosis.
-
Confidentiality must be maintained, and the patient’s consent is required before disclosing status to others.
-
The husband can then be encouraged to inform his spouse and take preventive measures.
-
Direct disclosure to others without consent breaches confidentiality.
-
Reporting to public health authorities happens per protocol but after patient counseling.
Rule out other options:
-
B. Inform wife directly: Breaches confidentiality.
-
C. Inform hospital administration: Not immediate priority.
-
D. Report immediately: Must follow counseling and consent procedures.
Clinical Tip:
Always start with patient-centered counseling before partner notification.
Q.77. A pregnant woman presents with a broken arm after being assaulted by her husband. During which time is domestic violence most likely to increase?
Correct Answer : C
Correct Answer:
C. During holidays
Explanation:
-
Domestic violence rates often increase during holidays due to stress, financial pressures, and increased time spent together.
-
Other stressful periods may contribute but holidays are well documented for higher risk.
-
Retirement or family visits may be factors but less commonly linked to acute spikes.
-
Violence typically happens in home settings, not at workplace.
Rule out other options:
-
A. After retirement: Possible long-term stressor but not immediate spike.
-
B. After family visits: Family presence may reduce or escalate violence, but less evidence for spike.
-
D. At work: Violence less likely in workplace setting.
Clinical Tip:
Always screen for domestic violence during holidays and stressful life events.
Q.78. A newly diagnosed child with type 1 diabetes mellitus is not given medication by the mother because she believes the child will become normal without treatment. What is the most likely psychological reaction exhibited by the mother?
Correct Answer : A
Correct Answer:
A. Denial
Explanation:
-
Denial is a common initial defense mechanism where a person refuses to accept reality or facts, especially after a distressing diagnosis.
-
The mother’s refusal to administer medication reflects denial of the child’s illness.
-
Neglect implies failure to provide necessary care, but here the behavior is rooted in denial.
-
Anxiety involves excessive worry but does not specifically explain refusal to treat.
Rule out other options:
-
B. Neglect: Could be secondary, but denial is the primary psychological process.
-
C. Anxiety: May coexist but does not explain refusal clearly.
Clinical Tip:
Recognize denial early and provide empathetic counseling to support families through diagnosis acceptance.
Q.79. A pregnant woman at 20 weeks gestation is diagnosed with bilateral ovarian abscess and refuses surgery. What is the appropriate action?
Correct Answer : A
Correct Answer:
A. Accept her decision and have her sign the refusal form
Explanation:
-
A competent pregnant woman has the right to refuse treatment even if it risks her or fetus’s health.
-
The physician should respect her autonomy and document refusal with a signed form.
-
Consent cannot be substituted by husband or relatives without her agreement.
-
Informing others without consent breaches confidentiality unless there is immediate risk to life and legal mandates.
Rule out other options:
-
B. Consent from husband: Not legally valid without patient consent.
-
C. Inform relative: Violates patient confidentiality.
-
D. Inform social/medical advisor: May be considered if patient lacks capacity or imminent risk exists, but not first step.
Clinical Tip:
Always document refusal thoroughly and provide counseling about risks.
Q.80. A patient is suspected to have meningitis. His son refuses to allow IV antibiotics to be administered. What is the most appropriate next step?
Correct Answer : B
Correct Answer:
B. Ask the patient for his opinion
Explanation:
-
If the patient is conscious and has decision-making capacity, only the patient has the legal and ethical right to make decisions regarding their care.
-
Family members cannot override a competent patient’s autonomy.
-
In life-threatening conditions like suspected meningitis, if the patient lacks capacity, emergency treatment can proceed in the patient’s best interest.
-
In this case, check if the patient is competent and involve them directly.
Rule out other options:
-
A. Respect the son's decision: Not appropriate if the patient is competent.
-
C. Proceed without consent: Only acceptable if the patient lacks capacity and there is imminent danger.
-
D. Refer to administration: Not the first step; this is a clinical ethics issue.
Clinical Tip:
Informed consent must always come from the patient, unless they are incapacitated—in which case, emergency treatment is justified.
Q.81. An elderly patient with end-stage cancer is experiencing unbearable pain. His family asks you to administer strong pain relief, knowing it may hasten death. What is the most appropriate action?
Correct Answer : B
Correct Answer:
B. Administer pain relief based on the principle of double effect
Explanation:
-
The principle of double effect allows the use of medications (e.g., opioids) to relieve severe pain even if it may unintentionally shorten life, as long as:
-
The intention is to relieve suffering (not to cause death).
-
The dose is proportional and medically justified.
-
-
This is ethically acceptable in palliative care and widely practiced.
-
Refusal or delay in treatment would be unethical if pain is not managed.
Rule out other options:
-
A. Refuse due to respiratory depression: Ignores ethical duty to relieve suffering.
-
C. Ask family to consent: Family cannot override medical judgment in this context; the focus is on patient needs.
-
D. Delay treatment and consult ethics: Not appropriate when prompt relief is needed.
Clinical Tip:
The goal of palliative care is comfort, not cure—use appropriate pain relief even if there's a foreseeable risk, provided your intent is to alleviate suffering.
Q.82. A doctor is considering giving a terminally ill patient a stronger painkiller, knowing it will relieve suffering but may have side effects such as sedation or respiratory depression. What type of ethical decision-making is being applied?
Correct Answer : C
Correct Answer:
C. Decision based on best interest (beneficence)
Explanation:
-
When administering stronger pain relief to alleviate suffering, the doctor applies beneficence—acting in the patient’s best interest.
-
This often involves weighing benefits vs. risks (such as sedation or respiratory depression).
-
This also falls under the principle of double effect when relieving pain may unintentionally hasten death.
-
Decisions are made ethically and medically, not by vote or family authority, unless the patient lacks capacity.
Rule out other options:
-
A. Decision by majority: Medical ethics are not democratic decisions.
-
B. Decision by family: Family may be consulted, but the patient's autonomy and best interest come first.
-
D. Decision by children: Not valid unless the patient lacks capacity and children are legal proxies.
Clinical Tip:
Use beneficence and proportionality when escalating pain management—your duty is to relieve suffering, even if there are secondary risks.
Q.83. Patient doesn't want to know the details, you are the surgeon appreciated that, but the anesthsic dr insist? a) Tell the patient the decision of the anasthesia dr. Answer: A
Correct Answer : A
Correct Answer:
A. Inform the patient of the anesthetist's decision
Explanation:
-
Autonomy includes the right to know as well as the right not to know.
-
However, each specialty involved in care (surgery, anesthesia) has a legal and ethical duty to obtain informed consent for its own procedures and risks.
-
If the anesthetist believes disclosure is necessary, the surgeon should communicate that to the patient, maintaining transparency while still respecting autonomy.
-
Patients can choose to decline details after being informed of that choice explicitly.
Rule out other options:
-
B. Respect the patient’s wish and withhold all details: This disregards the anesthetist’s legal duty to inform the patient of anesthesia-specific risks.
-
C. Proceed without telling the patient or anesthetist: Completely unethical and could be legally problematic.
-
D. Refer to ethics committee: Not necessary as this is best handled through direct communication and clarification with the patient.
Clinical Tip:
Patients have the right to waive information, but they must be made aware that other team members may need to explain specific risks for separate consents.
Q.84. A patient wears inappropriate clothing and speaks seductively to the doctor in a language that only the nurse understands. What is the most appropriate response by the doctor?
Correct Answer : C
Correct Answer:
C. Request the nurse who understands the language to stay during the consultation
Explanation:
-
The most professional and appropriate response is to maintain clear boundaries while avoiding confrontation.
-
Involving a chaperone or nurse who speaks the patient's language helps facilitate communication and protect both the doctor and the patient.
-
This approach preserves dignity and avoids escalation.
-
Direct confrontation or involving security without assessing the situation can be inappropriate and may worsen the doctor–patient relationship.
Rule out other options:
-
A. Shout at the patient: Unprofessional and escalates tension unnecessarily.
-
B. Call security: Premature and only appropriate if the patient becomes threatening or violent.
-
D. Ignore and continue: Fails to address boundary-testing behavior and communication issues.
Clinical Tip:
Always maintain professionalism and boundaries. Use a chaperone or interpreter when language or behavior creates discomfort or risk of misunderstanding.
Q.85. A 65-year-old patient with a facial lesion is diagnosed with melanoma based on biopsy. He is known to be non-compliant with previous treatments. What is the best approach to improve his compliance?
Correct Answer : C
Correct Answer:
C. Talk to him slowly, step by step, to help him understand
Explanation:
-
Non-compliance often results from fear, lack of understanding, or poor communication rather than defiance.
-
The best approach is empathetic, clear, and patient-centered communication, especially in older adults who may need more time to process information.
-
A step-by-step explanation in simple language builds trust, understanding, and improves compliance.
-
Showing off credentials or threatening the patient is counterproductive.
Rule out other options:
-
A. Tell him about the serious signs and symptoms: May scare rather than motivate; not effective alone.
-
B. Show him how brilliant you are: Arrogant and unhelpful.
-
D. Tell him he has no choice: Violates autonomy and likely worsens non-compliance.
Clinical Tip:
With elderly or non-compliant patients, use clear, calm, step-by-step explanations, involve caregivers if needed, and ensure they feel respected and empowered in decision-making.
Q.86. A patient has just been diagnosed with squamous cell carcinoma. As the attending physician, how should you break the news to him?
Correct Answer : D
Correct Answer:
D. Tell him gradually, in small steps over multiple conversations
Explanation:
-
Delivering bad news should be done sensitively, respecting the patient's emotional state, cultural background, and readiness.
-
The SPIKES protocol is a widely accepted method:
-
S – Setting up the interview
-
P – Assessing the patient's Perception
-
I – Obtaining the patient's Invitation
-
K – Giving Knowledge and information
-
E – Addressing Emotions
-
S – Strategy and Summary
-
-
Gradual disclosure allows patients to process the information, ask questions, and maintain emotional stability.
Rule out other options:
-
A. Reassurance, tell him not to worry: False reassurance can break trust and leave the patient unprepared.
-
B. Don’t tell him at all: Hiding a diagnosis violates patient autonomy.
-
C. Tell him immediately without delay: Dumping all information at once can overwhelm or traumatize the patient.
Clinical Tip:
Always assess how much the patient wants to know. Use empathetic language and pause regularly to check understanding and emotional response.
Q.87. A 38-week pregnant woman is diagnosed with intrauterine fetal demise (IUFD) with no identifiable cause. She is emotionally stable and asks about her management options. What is the most appropriate response by the doctor?
Correct Answer : B
Correct Answer:
B. Inform her that she still has time to go home and decide
Explanation:
-
In cases of IUFD at term, immediate induction is not always necessary unless there is maternal instability, infection, or bleeding.
-
Most mothers can be given a few days to process the diagnosis and make informed decisions about timing and mode of delivery.
-
This shared decision-making approach improves psychological outcomes.
-
Emotional support and clear communication are essential in these situations.
Rule out other options:
-
A. Induce labor immediately: Not wrong clinically, but not always necessary; may be psychologically distressing without preparation.
-
C. Emergency cesarean: Not indicated; no benefit to mother or fetus in IUFD.
-
D. Start oxytocin without discussion: Violates autonomy and is not ethically acceptable.
Clinical Tip:
In IUFD, give emotional space and counseling. Always support the patient with options and avoid rushing decisions unless medically indicated.
Q.88. A pregnant woman is diagnosed with HIV infection. What is the most appropriate advice to reduce the risk of transmitting the virus to her baby?
Correct Answer : B
Correct Answer:
B. Recommend cesarean section to reduce risk of vertical transmission
Explanation:
-
In HIV-positive pregnant women, especially those with detectable viral load, elective cesarean section at 38 weeks significantly reduces the risk of vertical (mother-to-child) transmission.
-
Additionally, antiretroviral therapy (ART) during pregnancy and postnatal ART for the newborn are key parts of management.
-
Breastfeeding is contraindicated in high-resource settings due to the risk of transmission unless no safe alternatives exist.
Rule out other options:
-
A. She can breastfeed: Incorrect in most settings; breastfeeding increases risk of HIV transmission.
-
C. No need to give the child vaccine after birth: False; routine vaccinations are important and recommended.
-
D. Reassure her she’s beautiful regardless of HIV: Irrelevant to clinical management, though emotional support is important.
Clinical Tip:
Always evaluate maternal viral load near term. If viral load >200 copies/mL, cesarean section is advised. Ensure ART adherence and plan neonatal prophylaxis.
Q.89. A 25-year-old pregnant woman at 33 weeks gestation is diagnosed with a ruptured ovarian cyst and is advised to undergo emergency surgery. The risks and procedure are clearly explained, but she refuses the operation. What is the most appropriate next step?
Correct Answer : A
Correct Answer:
A. Accept her refusal
Explanation:
-
A pregnant woman who is mentally competent has full legal and ethical right to refuse any medical treatment — even if it could affect the fetus.
-
Forcing treatment or seeking consent from others violates her autonomy and informed consent principles.
-
The role of the physician is to inform, counsel, and document — not to coerce.
-
If the refusal places her or the fetus at high risk, involve ethics or legal teams for support, but still respect her decision unless legally overridden by court (rare and extreme cases).
Rule out other options:
-
B. Force her into the OR: Gross violation of autonomy, bodily rights, and medical ethics.
-
C. Take consent from husband: Legally and ethically invalid unless the patient is incapacitated.
-
D. Take consent from a relative: Same as above — not applicable in a conscious, competent adult.
Clinical Tip:
Pregnant women retain full decision-making rights. Always ensure informed refusal is documented, and offer ongoing support and options. Autonomy applies regardless of gestational age.
Q.90. A baby diagnosed with an inguinal hernia requires urgent surgery. The mother has abandoned the child, and no immediate family member is available to give consent. Who should the pediatric surgeon approach for consent?
Correct Answer : C
Correct Answer:
C. Hospital Ethics Committee
Explanation:
-
When parents or legal guardians are unavailable, and urgent surgery is required, the Hospital Ethics Committee or legal guardian authority should be consulted to authorize consent.
-
This ensures the child’s best interests are protected while maintaining legal and ethical standards.
-
Proceeding without any consent, even in urgent cases, is only justified if delay would cause serious harm and no alternative exists.
Rule out other options:
-
A. Hospital social worker: Social workers assist but cannot legally consent for surgery.
-
B. The mother’s next of kin: Not always legally authorized; also may not be available.
-
D. Proceed without consent due to emergency: Only justified if surgery is life-saving and delay would harm the child.
Clinical Tip:
For minors without guardians, always involve ethics or legal committees early to guide decisions and avoid medico-legal issues.
Q.91. A woman presents with bruises caused by her husband’s physical violence. What is the most common underlying cause of violence against females globally?
Correct Answer : B
Correct Answer:
B. Cultural factors
Explanation:
-
Cultural norms and patriarchal beliefs often perpetuate and normalize violence against women.
-
Practices like male dominance, gender inequality, and acceptance of domestic violence in some cultures are major drivers.
-
Socioeconomic factors contribute but are generally secondary.
-
Genetic predisposition or medical illness do not directly cause intentional violence.
Rule out other options:
-
A. Socioeconomic factors: While poverty and unemployment can increase stress, culture plays a larger foundational role.
-
C. Genetic predisposition: No strong evidence linking genetics directly to domestic violence behavior.
-
D. Medical illness: Some psychiatric illnesses may influence behavior but are not the common cause.
Clinical Tip:
When managing domestic violence, understand cultural contexts and tailor your approach with sensitivity to beliefs, while offering protection and support.
Q.92. What determines the positive predictive value (PPV) of a diagnostic test in a population?
Correct Answer : C
Correct Answer:
C. Disease prevalence
Explanation:
-
Positive Predictive Value (PPV) is the probability that a person with a positive test actually has the disease.
-
PPV depends heavily on disease prevalence in the tested population.
-
As prevalence increases, PPV increases (more true positives).
-
Conversely, with lower prevalence, PPV decreases due to more false positives.
-
Sensitivity and specificity are intrinsic test properties but do not change with prevalence.
-
Negative Predictive Value (NPV) behaves inversely with prevalence.
Rule out other options:
-
A. Sensitivity: Probability that the test is positive if the disease is present; affects but does not determine prevalence.
-
B. Specificity: Probability that the test is negative if the disease is absent; also intrinsic, not prevalence-dependent.
-
D. None of the above: Incorrect as prevalence directly affects PPV.
Clinical Tip:
When screening rare diseases (low prevalence), positive results are more likely to be false positives—hence confirmatory testing is essential.
Q.93. A doctor explains to a mother the benefits of vaccination and the potential risks if her child does not receive the vaccine. What is this process called?
Correct Answer : B
Correct Answer:
B. Counseling
Explanation:
-
Counseling is the process of providing information, guidance, and support to help patients or caregivers make informed decisions.
-
In vaccination, counseling involves explaining benefits, risks, and possible consequences of refusal.
-
It differs from consent, which is the formal permission to proceed.
-
Coercion is forcing someone to act against their will, which is unethical.
-
Documentation is recording the process but is not the process itself.
Rule out other options:
-
A. Consent: Formal agreement, not the educational process.
-
C. Coercion: Unethical and involves pressure.
-
D. Documentation: Recording but not the interaction.
Clinical Tip:
Effective counseling improves vaccine acceptance and adherence by addressing fears and misinformation.
Q.94. A female patient requests a female healthcare provider to perform her physical examination. What principle should guide the doctor’s response?
Correct Answer : A
Correct Answer:
A. Respect for patient preference
Explanation:
-
Respecting a patient’s request for a provider of a specific gender demonstrates respect for patient autonomy and cultural or personal preferences.
-
This builds trust, increases comfort, and improves the clinical encounter.
-
Beneficence refers to acting in the patient’s best interest but doesn’t specifically address patient preferences.
-
Justice relates to fairness and equal treatment.
-
Confidentiality is about protecting private information.
Rule out other options:
-
B. Beneficence: Not directly related to respecting gender preference.
-
C. Justice: Ensuring fairness but not relevant here.
-
D. Confidentiality: Concerned with privacy of information, not provider gender.
Clinical Tip:
Always ask and respect patient preferences about the examiner’s gender, especially in sensitive exams, to promote dignity and trust.
Q.95. Parents refuse to vaccinate their child despite medical advice. What is the most appropriate next step?
Correct Answer : A
Correct Answer:
A. Involve social services
Explanation:
-
When parents refuse essential vaccinations that put the child at risk of serious disease, healthcare providers must protect the child's welfare.
-
If counseling fails, involving social services is appropriate to assess for neglect and ensure child safety.
-
Vaccinating without consent is legally and ethically problematic unless in emergencies or court order.
-
While counseling (Option D) is important, it is often the initial step; if refusal persists, social services should be involved.
Rule out other options:
-
B. Ethics department: Helpful for guidance but does not have authority to intervene.
-
C. Vaccinate without consent: Violates legal and ethical standards.
-
D. Counsel parents: Necessary but insufficient alone if refusal continues.
Clinical Tip:
Always document counseling sessions and refusals thoroughly. Know your local laws about mandatory vaccinations and child protection.
Q.96. A post-operative patient is found to have a retained infected surgical sponge in the abdomen. What is the most appropriate next step?
Correct Answer : B
Correct Answer:
B. Inform the patient, apologize, and arrange urgent removal
Explanation:
-
Ethical principles require full disclosure of medical errors to the patient.
-
Apologizing and involving the patient builds trust and respects their autonomy.
-
The retained sponge must be removed urgently to prevent complications.
-
Concealing the error violates professional ethics and legal standards.
Rule out other options:
-
A. Don't tell the patient: Unethical and can lead to legal consequences.
-
C. Inform ethics committee only: Ethics committee involvement may help but does not replace patient disclosure.
-
D. Wait and observe: Risky; infection and complications can worsen.
Clinical Tip:
Always follow open disclosure policies after medical errors. Prompt action and honest communication improve outcomes and reduce litigation risk.
Q.97. A patient requires limb amputation but refuses the procedure. What is the appropriate next step?
Correct Answer : B
Correct Answer:
B. Respect refusal and discharge the patient
Explanation:
-
Patients have the right to refuse treatment if they are competent and informed.
-
After explaining risks and benefits, the patient’s decision must be respected.
-
Referral might be considered if patient requests a second opinion, but refusal itself does not mandate referral.
-
Forcing treatment violates autonomy and legal rights.
-
Ethics committee involvement is usually reserved for conflicts or capacity issues.
Rule out other options:
-
A. Refer: Only if patient requests or capacity concerns arise.
-
C. Proceed regardless: Illegal and unethical without consent.
-
D. Ethics committee: Not mandatory unless there is a dispute or incapacity.
Clinical Tip:
Document thorough counseling and patient’s refusal carefully. Evaluate patient capacity to ensure refusal is informed and voluntary.
Q.98. An 8-year-old boy presents with measles. He was never vaccinated because his father believes vaccines are harmful. What is the appropriate next step?
Correct Answer : A
Correct Answer:
A. Explain the importance of vaccination to the father
Explanation:
-
The priority is to educate and counsel the parent about vaccine benefits and risks of refusal.
-
Measles is a serious vaccine-preventable disease, and understanding can improve compliance.
-
Immediate referral to child protection is usually reserved for neglect causing serious harm or risk.
-
Refusing treatment is unethical and illegal.
-
Treating the patient is mandatory, but counseling the guardian is essential to prevent future risk.
Rule out other options:
-
B. Child protection services: Reserved for neglect causing immediate danger or repeated harm.
-
C. Refuse treatment: Violates medical ethics and patient rights.
-
D. Treat and discharge: Treating alone without education misses prevention opportunity.
Clinical Tip:
Use effective communication strategies to address vaccine hesitancy, tailoring messages to cultural beliefs.
Q.99. A female patient with incomplete abortion refuses surgery, but her husband wants to sign the consent for her. Who has the legal right to give consent for the surgery?
Correct Answer : C
Correct Answer:
C. Patient herself
Explanation:
-
Competent adult patients have the right to consent or refuse treatment regardless of family or spouse wishes.
-
The husband or family cannot override the patient’s decision.
-
The doctor cannot give consent on behalf of the patient.
-
Respecting patient autonomy is a fundamental ethical and legal principle.
Rule out other options:
-
A. Husband: No legal authority to consent if the patient refuses.
-
B. Family: Same as husband, no overriding right.
-
D. Doctor: Can only recommend but not consent for surgery.
Clinical Tip:
Always assess patient capacity before accepting refusal; if competent, document the refusal and counsel thoroughly.
Q.100. During surgery, the common bile duct (CBD) is accidentally injured, but the patient is stable post-operatively. What should be your immediate next step?
Correct Answer : A
Correct Answer:
A. Inform the patient and apologize
Explanation:
-
Full disclosure of intraoperative complications is an ethical obligation.
-
Informing the patient promptly maintains trust and allows for shared decision-making about further management.
-
Concealing the injury is unethical and may lead to legal consequences.
-
Reporting to administration or ethics committee is important but does not replace informing the patient.
Rule out other options:
-
B. Do not tell the patient: Unethical, breaches patient autonomy.
-
C. Inform administration only: Insufficient without informing patient.
-
D. Wait: Delays necessary communication and decision-making.
Clinical Tip:
Always practice open disclosure after medical errors or complications to maintain patient trust and fulfill professional duties.
Q.101. A couple presents with post-coital bleeding and expresses sadness and worry. What is the most appropriate immediate response from the healthcare provider?
Correct Answer : A
Correct Answer:
A. Acknowledge their emotions and provide support
Explanation:
-
Emotional acknowledgment builds trust and facilitates better history-taking and management.
-
Understanding their concerns helps reduce anxiety and improves patient compliance.
-
Immediate invasive investigations without proper counseling may increase distress.
-
Referral to psychiatry is not first-line unless psychological issues are prominent.
Rule out other options:
-
B. Immediate invasive tests: Premature without proper clinical evaluation.
-
C. Reassure without listening: Dismissive, harms doctor-patient relationship.
-
D. Refer psychiatry: Not indicated as a first step.
Clinical Tip:
Always address patients’ emotional responses first before proceeding with diagnostic or treatment plans.
Q.102. During surgery for suspected appendicitis, the appendix appears normal but the surgeon decides to remove it anyway. What is the appropriate step regarding patient communication?
Correct Answer : A
Correct Answer:
A. Inform the patient about the intraoperative findings and rationale
Explanation:
-
Transparency about intraoperative findings respects patient autonomy.
-
Explaining the reason for removal of a normal-looking appendix helps build trust.
-
Concealing findings can harm the doctor-patient relationship.
-
Ethics committee involvement is not necessary unless there is dispute.
-
Waiting until pathology results delays disclosure and may confuse the patient.
Rule out other options:
-
B. Do not tell: Unethical and undermines informed consent.
-
C. Inform ethics committee only: Does not replace patient communication.
-
D. Wait for pathology: Delays important discussion and informed consent.
Clinical Tip:
Always discuss intraoperative findings and decisions with the patient post-operatively, ensuring clear and honest communication.
Q.103. An elderly patient with end-stage cancer has a valid Do Not Resuscitate (DNR) order signed by his wife. After the patient dies, his son is angry, questioning why resuscitation was not attempted. What is the appropriate response?
Correct Answer : A
Correct Answer:
A. Explain that the DNR respects the wife’s decision as the legal surrogate
Explanation:
-
The DNR order is a legally binding decision made by the patient or their authorized surrogate (in this case, the wife).
-
It’s important to communicate clearly about respecting the patient’s wishes.
-
Apologizing or ignoring the family’s emotions does not address legal and ethical obligations.
-
Reporting to administration may be necessary later, but first focus on family communication.
Rule out other options:
-
B. Apologize and promise resuscitation: Violates patient autonomy and DNR order.
-
C. Ignore son: Poor communication and empathy.
-
D. Inform administration: May be necessary, but does not replace direct family dialogue.
Clinical Tip:
Always discuss DNR orders with all family members beforehand if possible to reduce misunderstandings.
Q.104. You arrive late to the clinic, and the patient expresses anger about the delay. What is the most appropriate initial response?
Correct Answer : A
Correct Answer:
A. Explore the reasons behind the patient’s anger
Explanation:
-
Understanding the cause of anger helps address the patient’s concerns effectively.
-
Ignoring anger or dismissing the patient worsens the doctor-patient relationship.
-
Simply telling the patient to be patient is dismissive.
-
Apologizing is important but ending the consultation quickly neglects the patient’s emotional needs.
Rule out other options:
-
B. Ignore anger: May escalate conflict and reduce satisfaction.
-
C. Tell patient to be patient: Disrespectful and dismissive.
-
D. Apologize but end quickly: Does not fully address patient concerns.
Clinical Tip:
Always acknowledge and explore patient emotions first to build trust and improve communication.
Q.105. A child with apparent intellectual disability is brought by parents who insist on taking him home despite medical concerns. What is the most appropriate next step?
Correct Answer : B
Correct Answer:
B. Consult the hospital ethics committee
Explanation:
-
When parents insist on decisions that may not be in the child’s best interest, involving the ethics committee helps guide appropriate care and protects the child’s welfare.
-
Immediate involvement of child protection services may be premature without assessing the situation thoroughly.
-
Blindly respecting parents’ decision could neglect the child’s rights and needs.
-
Discharging without follow-up may endanger the child.
Rule out other options:
-
A. Call child protection immediately: May be necessary later but not first step.
-
C. Respect parents without question: Risks harm if child’s needs are unmet.
-
D. Discharge without follow-up: Unsafe and unethical.
Clinical Tip:
Always involve an ethics committee in complex cases where patient welfare and family wishes conflict.
Q.106. An elderly patient with end-stage cancer is in unbearable pain. The family requests stronger painkillers. What is the most appropriate action?
Correct Answer : A
Correct Answer:
A. Administer stronger analgesics to relieve pain
Explanation:
-
Managing severe pain in terminal cancer patients is a priority to improve quality of life.
-
The principle of double effect supports using stronger analgesics even if it may hasten death, provided the intent is pain relief.
-
Refusing stronger analgesics or delaying treatment is unethical and causes unnecessary suffering.
-
Non-pharmacological measures can complement but not replace effective analgesia in severe pain.
Rule out other options:
-
B. Refuse stronger painkillers: Inappropriate; fear of addiction should not limit pain relief in terminal illness.
-
C. Non-pharmacological only: Insufficient for severe cancer pain.
-
D. Delay treatment: Causes unnecessary patient suffering.
Clinical Tip:
In end-stage cancer, prioritize pain control with appropriate opioids while monitoring side effects and patient comfort.
Q.107. A patient does not want to know the surgical details, and the surgeon respects this. However, the anesthesiologist insists on informing the patient about anesthesia risks. What is the best approach?
Correct Answer : B
Correct Answer:
B. Inform the patient that the anesthesiologist wants to share their opinion and ask if they agree
Explanation:
-
Patient autonomy must be respected, including their wish not to know details.
-
The anesthesiologist’s concerns should be communicated sensitively, seeking the patient’s permission.
-
Forcing information on the patient or withholding communication harms trust.
-
Collaboration and respect among the care team improve patient-centered care.
Rule out other options:
-
A. Tell without consent: Violates patient autonomy.
-
C. Prevent anesthesiologist: Undermines teamwork and patient safety.
-
D. Ignore anesthesiologist: Risks missing important consent issues.
Clinical Tip:
Always respect patient preferences while ensuring informed consent is obtained in a sensitive, collaborative manner.
Q.108. A male patient who previously came for infertility assessment is found to be HIV positive. Who should be informed first about the test result?
Correct Answer : A
Correct Answer:
A. Inform the patient first
Explanation:
-
Confidentiality and patient autonomy require that the patient is informed about their HIV status first.
-
Disclosure to others (including spouse) should be done with the patient’s consent unless public health laws mandate otherwise.
-
Informing the wife without the patient’s knowledge breaches confidentiality.
-
Public health authorities are informed per legal requirements but only after informing the patient.
Rule out other options:
-
B. Inform wife first: Breaches patient confidentiality.
-
C. Inform both together: Requires patient’s agreement.
-
D. Inform authorities first: Patient must be informed first unless mandated otherwise.
Clinical Tip:
Always maintain patient confidentiality and offer counseling and support before disclosure to third parties.
Q.109. After explaining the management plan and exploring the patient’s emotions, the doctor reviews the main points again to ensure understanding. What communication skill is the doctor demonstrating?
Correct Answer : A
Correct Answer:
A. Summarization
Explanation:
-
Summarization involves reviewing key information to confirm understanding and reinforce important points.
-
Reflection focuses on mirroring the patient’s feelings or statements.
-
Clarification is asking for or providing more detailed information.
-
Empathy is recognizing and responding to patient emotions.
Rule out other options:
-
B. Reflection: Involves echoing patient emotions, not repeating facts.
-
C. Clarification: Seeks to explain or understand specifics, not general review.
-
D. Empathy: Concerned with feelings, not restating content.
Clinical Tip:
Use summarization at the end of consultations to ensure patient understanding and address any confusion.
Q.110. A patient with complete abortion is tearful while waiting for admission. Her husband appears emotionally distressed. What is the most appropriate initial approach by the healthcare team?
Correct Answer : A
Correct Answer:
A. Explore and reflect their emotions
Explanation:
-
Addressing emotional distress by exploring and reflecting feelings helps patients and families feel understood and supported.
-
Ignoring or dismissing emotions can worsen psychological distress.
-
Reassurance without listening may feel dismissive.
-
Referral to psychiatry might be necessary later but should follow initial emotional support.
Rule out other options:
-
B. Ignore emotions: Unempathetic and harmful.
-
C. Reassure without listening: Does not address underlying feelings.
-
D. Immediate psychiatric referral: Premature without initial emotional support.
Clinical Tip:
Use active listening and reflection to provide immediate emotional support in acute distress situations.
Q.111. A fetal death is discovered during a routine antenatal visit. The mother begins to cry and her husband appears visibly distressed. What is the most appropriate action as a physician?
Correct Answer : C
Correct Answer:
C. Explore, recognize their emotions, and find solutions
Explanation:
-
Option C includes a comprehensive approach:
-
Explore: Ask open-ended questions to allow expression.
-
Recognize emotions: Acknowledge and name what they may be feeling (sadness, guilt, confusion).
-
Find solutions: Help navigate next steps (e.g., support services, future care plan).
-
-
While B is a strong emotional support step, C reflects the complete physician responsibility — not just to comfort, but also to guide and plan next actions.
Rule out other options:
-
A. Too early and inappropriate — discussing malformations now is insensitive.
-
B. Correct but not complete — lacks action plan for care or guidance.
-
D. Empathy alone is not enough — does not provide emotional or medical support.
Clinical Tip:
In emotionally charged moments, use the SPIKES protocol:
Set up the conversation
Perception – assess what the patient understands
Invitation – ask how much they want to know
Knowledge – deliver the information
Emotions – respond with empathy
Strategy & Summary – discuss next steps
Q.112. A patient with a Do-Not-Resuscitate (DNR) order presents in respiratory distress. The surgeon believes that a debulking surgery might improve the patient’s condition, but it carries a high risk and could be fatal. What is the most appropriate next step?
Correct Answer : B
Correct Answer:
B. Take high-risk surgical consent
Explanation:
-
A DNR order refers specifically to not initiating CPR or advanced resuscitation in the event of cardiac or respiratory arrest.
-
It does not prohibit active treatment or surgery that might help the patient if the patient is informed and consents.
-
In this case, the surgery may offer benefit, so proceeding with high-risk consent (fully disclosing the risks and obtaining informed consent) is ethically and legally appropriate.
Rule out other options:
-
A. Misinterprets DNR; it does not mean “do not treat” — only no CPR.
-
C. Referring to palliative care may be needed later, but not before offering the surgical option if beneficial.
-
D. Ethics consultation is unnecessary if informed consent is obtained directly from the patient.
Clinical Tip:
DNR ≠ Do Not Treat. It only limits resuscitation measures, not therapeutic or life-improving interventions. Always reassess decision-making capacity and involve the patient or surrogate in high-risk decisions.
Q.113. You arrive late to your clinic, and your patient becomes visibly upset and begins shouting. What is the most appropriate initial response?
Correct Answer : B
Correct Answer:
B. Acknowledge the patient's anger and listen to his concerns
Explanation:
-
When a patient is angry, the first and most effective step is to acknowledge and validate their emotions. This approach de-escalates the situation and opens communication.
-
Attempting to justify, ignore, or delegate the interaction without addressing the patient’s emotional state may escalate the conflict and damage trust.
Rule out other options:
-
A. Justifying the delay without first acknowledging the patient's emotions may seem dismissive.
-
C. Ignoring the patient's anger is unprofessional and can worsen the situation.
-
D. Asking someone else to handle it avoids responsibility and undermines doctor-patient rapport.
Clinical Tip:
Acknowledging emotions is the cornerstone of de-escalation. Simple statements like “I understand you're upset. Let's talk about it.” can significantly reduce tension and maintain therapeutic trust.
Q.114. A patient has been diagnosed with a condition but says she does not believe the diagnosis is correct and requests to see another doctor. What is the most appropriate action?
Correct Answer : A
Correct Answer:
A. Provide her with the medical report as it is her right
Explanation:
-
Patients have the right to autonomy in medical decision-making, which includes the right to seek a second opinion.
-
Providing her report empowers her decision-making and supports transparency and trust.
-
This action is ethically and legally appropriate, promoting patient-centered care.
Rule out other options:
-
B. Referring her may be a follow-up step but is not the immediate or most appropriate first action.
-
C. There's no ethical dilemma here requiring committee involvement; the situation is straightforward.
-
D. Trying to convince the patient against their will undermines autonomy and informed decision-making.
Clinical Tip:
Always respect patient autonomy. If a patient requests a second opinion, never take it personally. Providing access to their records is a legal and ethical obligation in almost all medical systems.
Q.115. A 30-year-old married woman visits the clinic for insertion of an intrauterine device (IUD) for contraception. She is well-informed, not pregnant, and medically fit. Her husband is currently overseas and not contactable. Who must provide consent for the procedure?
Correct Answer : A
Correct Answer:
A) Wife
Explanation:
In medical ethics and legal practice, the person undergoing the procedure must provide informed consent. In the case of IUD insertion:
-
Only the woman’s consent is required, as it is her body and autonomy that is affected.
-
Requiring the husband's or anyone else's consent would violate principles of bodily autonomy and patient rights.
Even in a marital relationship, each individual retains the right to make decisions regarding their own reproductive health.
Ruling Out Other Options:
-
B) Husband – Not acceptable; he is not undergoing the procedure.
-
C) Mutual consent from both – Ethically and legally incorrect. While mutual discussion may be encouraged for family planning, consent must be from the person undergoing the procedure.
-
D) Legal guardian – Only applies if the patient is mentally incompetent or legally incapacitated, which is not the case here.
Clinical Tip:
Informed consent must come from the individual receiving the intervention.
Respect for patient autonomy is a foundational principle of bioethics, particularly in reproductive and contraceptive decisions.
Q.116. A newborn child is abandoned in the hospital by his mother after delivery, reportedly as a result of an out-of-marriage relationship. The infant is currently in the neonatal unit and diagnosed with an inguinal hernia that requires urgent surgical repair. The pediatric surgeon is ready, but there is no legal guardian available to sign the consent. What is the most appropriate next step?
Correct Answer : A
Correct Answer:
A) Review with the hospital ethics committee or obtain court/legal authorization
Explanation:
In cases where a minor lacks a legal guardian and an urgent medical procedure is needed, the best ethical and legal course is:
-
Involve the hospital's ethics committee, or
-
Obtain emergency court/legal permission, especially if the procedure is not immediately life-saving but medically indicated.
This balances the child's best interest with legal safeguards.
-
If the hernia is not incarcerated or strangulated, it’s not an emergency requiring immediate action without consent.
-
Therefore, ethical/legal clearance is required before proceeding.
Ruling Out Other Options:
-
B) Do the surgery without consent
Not appropriate unless it's a life-threatening emergency (e.g., hernia is strangulated). Otherwise, legal/ethical oversight is necessary. -
C) Refuse to do it as no one to sign
Unethical. The child still deserves care. The healthcare system must act in the child's best interest through appropriate channels. -
D) Wait until the child is placed in an orphanage or foster system
This may delay care unnecessarily. The child's current status still mandates timely action via existing hospital/legal pathways.
Clinical Tip:
In abandoned or unaccompanied minors, you must act in the best interest of the child, while also respecting legal and institutional processes.
For non-life-threatening but necessary procedures, always involve the hospital ethics committee or child protection services.
Q.117. While taking a history from a 45-year-old male patient, he interrupts you and begins expressing his personal opinion about the cause of his illness and what he believes is the diagnosis. How should you respond?
Correct Answer : B
Correct Answer:
B) Let him finish his point of view
Explanation:
Active listening is a critical part of the physician-patient relationship. When a patient interrupts to express their understanding or belief about their illness, it's important to:
-
Allow them to speak fully.
-
Show respect and empathy.
-
Understand their ideas, concerns, and expectations (ICE approach).
By doing so, you establish trust, avoid confrontation, and gather valuable psychosocial context that may guide further questioning and management.
Ruling Out Other Options:
-
A) Refer him to another doctor
Not appropriate; dismissive and unprofessional. It undermines continuity of care and patient rapport. -
C) Apologize to him and start with closed-ended questions
This may seem structured, but it cuts off the patient’s narrative and may cause them to feel unheard. -
D) Tell him to wait until you finish asking your questions
Disrespectful and may damage the therapeutic relationship. It ignores patient-centered communication.
Clinical Tip:
Use the "ICE" method (Ideas, Concerns, Expectations) during history-taking to explore a patient’s perspective.
This not only helps in accurate diagnosis but also improves patient satisfaction and adherence.
Q.118. A patient is scheduled for elective surgery. During the consent process, the surgeon begins explaining the benefits and risks of the operation, but the patient interrupts and says he does not want to hear about the possible complications. However, the anesthesiologist insists that the patient must be informed about the potential anesthesia-related risks. What is the most appropriate next step?
Correct Answer : C
Correct Answer:
C) Inform the patient about the complications as the anesthesiologist advised
Explanation:
Informed consent is a legal and ethical requirement before any medical or surgical intervention. It includes:
-
Nature of the procedure
-
Benefits
-
Alternatives
-
Risks and potential complications
Even if a patient is hesitant or uncomfortable, physicians are legally and ethically obligated to explain potential complications — especially those related to anesthesia, which can be significant.
-
Respectfully but clearly communicating the risks is not optional; skipping this step would result in incomplete consent, which could have legal consequences.
Ruling Out Other Options:
-
A) Cancel the operation
Unnecessary at this stage. The goal is to complete the consent process properly, not to abandon care. -
B) Get another anesthesiologist
Irrelevant. The issue is not with the provider but with the need to uphold proper informed consent. -
D) Let the anesthesiologist deal with the patient
While the anesthesiologist may explain anesthesia-specific risks, the primary physician is also responsible for ensuring complete informed consent is obtained. Passing the responsibility entirely is inappropriate.
Clinical Tip:
Informed consent is not just a formality — it's a patient's right.
Even if patients show resistance, it’s your duty to gently but firmly communicate key risks, particularly those with serious consequences like anesthesia-related events.
Q.119. A 40-year-old male smoker visits the clinic and says, “Doctor, I’ve decided I want to quit smoking and I need help.” What is the most appropriate next step in counseling based on the stages of change model?
Correct Answer : C
Correct Answer:
C) Preparation
Explanation:
This patient has already expressed a desire to quit smoking and is seeking assistance. According to the Transtheoretical Model (Stages of Change), this places him in the Preparation stage, where the patient:
-
Has decided to change behavior
-
Is planning to take action soon (usually within 30 days)
-
May begin setting a quit date and discussing strategies
In this stage, your role is to support the patient in planning: offer pharmacotherapy options (e.g., nicotine replacement, varenicline), behavioral support, and set a specific quit date.
Ruling Out Other Options:
-
A) Precontemplation – Patient is not even thinking about quitting. Incorrect, as this patient is actively asking to quit.
-
B) Contemplation – Patient is considering quitting but hasn’t made a decision. Also incorrect in this case.
-
D) Action – This is the stage after quitting has already occurred. The patient hasn’t yet quit, so this is premature.
Clinical Tip:
The "Stages of Change" model helps guide smoking cessation. When a patient is in the preparation stage, always work with them to set a quit date and offer tailored pharmacologic and behavioral support.
Q.120. A pregnant woman who smokes wants to quit. What is the best and safest method to help her stop smoking during pregnancy?
Correct Answer : A
Correct Answer:
A) Behavioral therapy
Explanation:
During pregnancy, the primary goal is to avoid any potential harm to the fetus. Therefore:
-
Behavioral therapy (counseling, support groups, cognitive-behavioral therapy) is the first-line and safest approach.
-
Pharmacologic agents like bupropion and nicotine replacement therapy (NRT) have limited safety data and are generally avoided unless benefits outweigh risks.
-
Electronic cigarettes and other smoking alternatives are not recommended due to uncertain safety and potential harmful effects.
Ruling Out Other Options:
-
B) Bupropion – Classified as FDA pregnancy category C; insufficient safety data, not routinely recommended in pregnancy.
-
C) Electronic cigarettes – Potentially harmful chemicals and unknown fetal risks; not recommended.
-
D) Alternative smoking products (herbal cigarettes) – Still involve inhalation of harmful substances; not safe in pregnancy.
Clinical Tip:
Always prioritize behavioral counseling for pregnant smokers.
If pharmacotherapy is considered, it should only be after careful risk-benefit analysis and under specialist supervision.
Q.121. A 67-year-old man was admitted after a fall with hip and knee pain but no fractures. A Foley catheter was inserted on day 1 due to immobility. After 5 days, he was discharged to a nursing home with the Foley catheter still in place. He now presents with fever and a urine analysis showing pyuria and bacteriuria. Which of the following measures best prevents catheter-associated urinary tract infection (CAUTI)?
Correct Answer : D
Correct Answer:
D) Daily review for the need of Foley catheter
Explanation:
The most effective prevention for catheter-associated urinary tract infections is to avoid unnecessary catheterization and to remove the catheter as soon as it is no longer needed. Daily review of the catheter need reduces infection risk.
-
Prophylactic antibiotics have not been shown to reduce CAUTI and contribute to antibiotic resistance.
-
Increasing IV fluids helps maintain hydration but does not directly prevent CAUTI.
-
Routine catheter changes do not prevent infection and may increase trauma and risk.
Ruling Out Other Options:
-
A & B) Prophylactic antibiotics
Overuse leads to resistance; no clear benefit in preventing CAUTI in patients with long-term catheters. -
C) Increase IV fluid intake
Good for hydration but insufficient alone to prevent infection.
Clinical Tip:
CAUTI prevention depends on minimizing catheter use.
Implement daily assessments to remove catheters as early as possible to reduce infection risk.
Q.122. A 12-year-old child’s parent has a chronic illness. How is the child most likely to respond behaviorally?
Correct Answer : D
Correct Answer:
D) Displays refractory behavior such as fussiness and occasional violence
Explanation:
Children around 12 years old usually understand the basics of disease and treatment but may struggle emotionally with the stress of a parent’s illness. Common reactions include:
-
Refractory behavior: fussiness, acting out, and sometimes aggression.
-
Emotional responses such as isolation, guilt, anxiety, and worry about the parent's health are frequent.
-
These behaviors are coping mechanisms to manage stress and uncertainty.
Ruling Out Other Options:
-
A) Can't comprehend disease, treatment, prognosis
Incorrect; by age 12, children generally have adequate cognitive capacity to understand basic illness concepts. -
B) Depression and withdrawal
Possible but not the only or primary response; children can also express distress through acting out. -
C) Forget
Unlikely; children are typically very aware of parental illness, though responses vary.
Clinical Tip:
When dealing with children of ill parents, watch for behavioral changes like irritability and aggression — these may signal emotional distress and need for psychological support.
Q.123. A baby is delivered at 24 weeks gestation. The mother, who is a physician, requests that no resuscitation be performed on the newborn. What is the most appropriate next step?
Correct Answer : A
Correct Answer:
A) Ignore the mother’s wishes and proceed with resuscitation
Explanation:
At 24 weeks gestation, the baby is considered extremely premature but may have a reasonable chance of survival with intensive neonatal care, depending on local guidelines and resources.
-
In cases where the infant is potentially viable, the healthcare team has a duty to provide life-saving care unless there is a valid Do Not Resuscitate (DNR) order or clear evidence that resuscitation would be futile or against the infant's best interest.
-
Maternal refusal of resuscitation for a viable infant is ethically complex but generally not binding if the baby’s survival chances are significant and resuscitation aligns with medical standards.
-
Social services involvement is only indicated if there is concern about neglect or abuse, which is not applicable here.
-
Ethical principles prioritize the best interest of the newborn, which typically requires resuscitation unless medically futile.
Ruling Out Other Options:
-
B) Tell social services
Not warranted unless there is suspicion of neglect or harm; parental refusal alone does not justify this. -
C) Respect her wishes
While parental input is important, it is not absolute when a newborn’s life is at stake and survival is likely.
Clinical Tip:
In extremely preterm births, follow local neonatal resuscitation guidelines and act in the infant’s best interest.
When parental wishes conflict with standard care, involve ethics committees and multidisciplinary teams for support.
Q.124. A patient diagnosed with Ebola virus disease (EVD) is admitted to the hospital but insists on leaving against medical advice. What is the most appropriate immediate action?
Correct Answer : A
Correct Answer:
A) Call security to prevent the patient from leaving
Explanation:
Ebola is a highly contagious and life-threatening infectious disease with serious public health implications. Patients diagnosed with Ebola must be isolated and managed in specialized settings to prevent transmission.
-
Allowing a patient with Ebola to leave the hospital unmonitored poses a significant risk to the community and healthcare workers.
-
Immediate measures to prevent the patient from leaving include involving hospital security and infection control teams.
-
While referral to infectious diseases is important, it does not override the urgent need to contain the patient.
-
Signing an AMA form is inappropriate in this context because public health safety overrides patient autonomy.
Ruling Out Other Options:
-
B) Refer to infectious department
This is part of routine care but does not address the urgent risk if the patient is attempting to leave. -
C) Sign AMA form
Ebola infection is a public health emergency, and patients cannot legally leave without containment measures.
Clinical Tip:
For highly infectious diseases, public safety and containment take precedence over patient autonomy.
Hospitals must have protocols involving security and infection control for such cases.
Q.125. An internal medicine resident performs a diabetic foot examination but forgets to close the curtains, exposing the patient’s abdomen. Which ethical principle has the resident most likely violated?
Correct Answer : A
Correct Answer:
A) Privacy
Explanation:
Privacy refers to the patient’s right to control access to their personal body and information. Failing to close curtains and exposing parts of the patient’s body without consent is a direct violation of their physical privacy.
-
Dignity is related but broader, involving respect and treating patients as valued individuals; although dignity may be affected, the immediate violation here is of privacy.
Ruling Out Other Options:
-
B) Dignity
While dignity is important and may be impacted, the core ethical breach here is failure to maintain the patient’s privacy.
Clinical Tip:
Always ensure adequate measures (e.g., closing curtains, draping) to protect patient privacy during physical examinations.
Q.126. A woman is brought to the hospital by the police with multiple bruises. How should you best approach her?
Correct Answer : D
Correct Answer:
D) Gently console her, acknowledge she may know the cause of bruises, then begin the examination
Explanation:
When managing a patient with possible abuse:
-
Establish trust and rapport by approaching the patient gently and respectfully.
-
Recognize that the patient may be fearful or ashamed but often knows the cause of injuries.
-
Consoling the patient helps to reduce anxiety and encourages cooperation.
-
Avoid forcing explanations or assuming denial; provide a safe, supportive environment.
Ruling Out Other Options:
-
A) Call social services immediately
Important but after initial clinical assessment and ensuring patient safety. -
B) Do not examine without husband
Never require the presence of a potential abuser or family member who may intimidate the patient. -
C) Enter with a large smile and immediately start exam
Inappropriate and may make the patient uncomfortable or fearful.
Clinical Tip:
In cases of suspected abuse, use a trauma-informed approach: be empathetic, nonjudgmental, and ensure privacy and safety.
Q.127. A stroke patient with hemiplegia develops first-degree bedsores on day 2. The physician verbally instructs the nurse to reposition the patient every hour, but the nurse fails to document the order. Three days later, the patient develops infected grade 3 bedsores and fever. Where was the primary defect?
Correct Answer : A
Correct Answer:
A) Inadequate communication between the doctor and the nurse
Explanation:
Effective communication in healthcare is critical to patient safety:
-
Verbal orders without documentation can lead to misinterpretation or omission of care instructions.
-
The lack of written orders is a common source of errors and compromises continuity of care.
-
Proper documentation ensures clear responsibility and timely intervention.
While nurse negligence or physician follow-up may contribute, the root cause here is the failure to properly communicate and document the care plan.
Ruling Out Other Options:
-
B) Nurse negligence
The nurse may not have been aware of the importance due to no written order. -
C) Lack of physician’s follow-up
Important but the primary issue was unclear communication. -
D) Patient’s underlying condition
Bedsores are preventable with proper care despite patient immobility.
Clinical Tip:
Always document all orders in patient records to ensure clear communication and accountability.
Q.128. A pregnant woman refuses a cesarean section (CS) that is medically indicated to save the baby. What is the most appropriate action?
Correct Answer : B
Correct Answer:
B) Respect her wishes
Explanation:
-
Respecting patient autonomy is a fundamental ethical principle; competent adults have the right to refuse treatment, even if it risks harm to the fetus.
-
The healthcare provider must inform the patient thoroughly about the risks to the baby and mother but cannot force treatment against her will.
-
Consent from the husband or others cannot override the mother’s decision.
-
Referral to another hospital may be an option but does not override the initial respect for autonomy.
-
Performing surgery without consent is legally and ethically impermissible except in emergencies where the patient is unable to consent.
Ruling Out Other Options:
-
A) Refer the patient
Can be considered if the patient desires but is not the immediate solution. -
C) Take husband consent
Legally invalid; patient’s consent is mandatory. -
D) Do vaginal delivery regardless
May not be safe or feasible without consent for CS. -
E) Do CS against her wish
Violates patient autonomy and legal rights.
Clinical Tip:
Always provide clear counseling on risks and benefits and document the patient’s informed refusal carefully.
Q.129. An elderly patient admitted with COPD exacerbation becomes aggressive and disoriented at night. What should the nurse do first?
Correct Answer : D
Correct Answer:
D) Elevate the head of the bed, apply nasal oxygen, and try to orient her to place and time
Explanation:
-
The patient’s agitation and disorientation are likely due to hypoxia or hypercapnia from COPD exacerbation.
-
The priority is to correct hypoxia by elevating the head to improve breathing and administering supplemental oxygen.
-
Reorientation helps reduce delirium symptoms.
-
Sedatives like lorazepam can worsen respiratory depression and should not be first-line.
-
Physical restraints can increase agitation and risk injury and are a last resort.
-
Family presence may help but is secondary to stabilizing oxygenation and orientation.
Ruling Out Other Options:
-
A) Call doctor for lorazepam
Sedatives may worsen respiratory status and delirium. -
B) Restrain patient
Restraints can exacerbate agitation and cause harm. -
C) Call family
Helpful but not immediate priority over oxygenation and positioning.
Clinical Tip:
In acute COPD exacerbations with agitation, always assess and correct oxygenation and ventilation first before sedation or restraints.
Q.130. A 7-year-old sick child is not compliant with his medication. As a doctor, what is your best initial approach?
Correct Answer : A
Correct Answer:
A) Explain to the parents the importance of medication compliance
Explanation:
-
Medication noncompliance in children is often related to lack of understanding or difficulty in administration.
-
The first step is to educate and counsel the parents about the critical importance of adhering to the prescribed treatment.
-
Involving the child in the plan (option C) is important but secondary to ensuring the parents understand compliance, especially in younger children.
-
Calling the police or Ethics Committee is unnecessary and inappropriate unless there is suspected neglect or abuse.
Ruling Out Other Options:
-
B) Call the police
Not justified without evidence of neglect or abuse. -
C) Involve parents and child
Good step but education on compliance is the priority. -
D) Refer to Ethics Committee
Not applicable unless ethical or legal issues arise.
Clinical Tip:
Effective communication with caregivers is key to improving pediatric medication adherence.
Q.131. A woman who conceived after 10 years of trying is now 10 weeks pregnant. She presents to the ER with abdominal pain and vaginal bleeding. The doctor finds no fetal heartbeat and diagnoses abortion. The couple is shocked. What should the doctor do first?
Correct Answer : A
Correct Answer:
A) Show sympathy and provide emotional support
Explanation:
-
The first and most crucial step after delivering bad news is to provide compassionate communication and emotional support.
-
The patient and family need time to process the loss before proceeding with further management or investigations.
-
After emotional stabilization, the patient can be prepared for medical or surgical management (admission and evacuation).
-
Investigating the cause is important but not the immediate priority in the emergency setting.
Ruling Out Other Options:
-
B) Admit for evacuation
Important but should follow emotional support and informed consent. -
C) Search for cause
Can be done later after stabilization and management of the abortion.
Clinical Tip:
Deliver bad news with empathy; use the “SPIKES” protocol (Setting, Perception, Invitation, Knowledge, Empathy, Strategy).
Q.132. A 75-year-old terminal lung cancer patient, with impaired decision-making capacity, is proposed for debulking surgery to relieve breathing difficulty. The surgery may improve quality of life but carries risks. What should the doctor do?
Correct Answer : C
Correct Answer:
C) Search for an alternative decision-maker to provide consent
Explanation:
-
When a patient lacks capacity to make informed decisions, the physician must seek a legally authorized surrogate or alternative decision-maker (family member, legal guardian) to give consent.
-
Proceeding without valid consent violates ethical and legal principles.
-
Attempting to obtain consent from an incapacitated patient is inappropriate.
Ruling Out Other Options:
-
A) Go ahead surgery
Not permissible without valid consent or surrogate approval unless in emergency and life-saving. -
B) Informed consent from patient
Not possible if patient lacks capacity.
Clinical Tip:
Assess decision-making capacity early; if impaired, identify and involve the legally authorized surrogate promptly.
Q.133. A patient is admitted with a hernia that requires surgery. Surgery is recommended now. What should you do if: It is an emergency? It is not an emergency?
Correct Answer : C
Correct Answer:
-
If emergency: C) Perform surgery without consent
-
If not emergency: A) Refuse surgery without consent
Explanation:
-
In an emergency, when delay may cause harm or death and the patient cannot give consent, surgery can be performed without consent under the principle of implied consent to save life or prevent serious harm.
-
In a non-emergency setting, surgery must not be performed without informed consent, respecting patient autonomy and legal requirements.
-
Consulting the ethics committee may help in complex cases but does not replace consent.
Ruling Out Other Options:
-
A) Refuse surgery without consent
Correct in non-emergencies, not in emergencies. -
B) Consult ethics committee
Useful but may delay necessary emergency care. -
D) Unknown
Cannot evaluate.
Clinical Tip:
Always assess urgency: in emergencies, prioritize saving life; otherwise, obtain informed consent.
Q.134. A child with mild flu symptoms is brought by his mother for scheduled vaccinations. The mother wants to postpone because the child is sick. The child is active with only mild symptoms. What should you do?
Correct Answer : C
Correct Answer:
C) Explain to the mother about vaccination safety and proceed to vaccinate
Explanation:
-
Mild illness (e.g., low-grade fever, mild respiratory symptoms) is not a contraindication to vaccination.
-
Delaying vaccines unnecessarily can increase risk of vaccine-preventable diseases.
-
The physician should educate the mother on the safety of vaccinating during mild illness and proceed with vaccination.
-
Postponing is indicated only for moderate to severe illness.
Ruling Out Other Options:
-
A) Reschedule in 2 weeks
Unnecessary if illness is mild; leads to vaccine delays. -
B) Ignore mother’s concerns
Poor communication; better to explain and reassure.
Clinical Tip:
Mild illness without fever or systemic symptoms should not delay vaccination.
Q.135. A pregnant woman wants to quit smoking. What is the best initial method to help her quit?
Correct Answer : A
Correct Answer:
A) Cognitive Behavioral Therapy (CBT)
Explanation:
-
Behavioral interventions such as CBT are the first-line and safest approach for smoking cessation during pregnancy.
-
Nicotine Replacement Therapy (NRT) may be considered only if behavioral therapy fails, and under medical supervision.
-
Bupropion is generally not recommended during pregnancy due to limited safety data.
Ruling Out Other Options:
-
B) Nicotine Replacement Therapy
Used cautiously, but not first-line. -
C) Bupropion
Limited evidence in pregnancy, potential risks.
Clinical Tip:
Always prioritize non-pharmacological methods like counseling and CBT for smoking cessation in pregnancy.
Q.136. A 34-year-old pregnant woman, gravida 4 para 3, currently at 32 weeks gestation, has never attended antenatal care (ANC) visits. What is the most likely cause?
Correct Answer : B
Correct Answer:
B) Ignorance about importance of ANC
Explanation:
-
Lack of awareness or knowledge about the importance of ANC is a common cause of missed visits, especially in low-resource or rural settings.
-
Financial barriers or fear can contribute but are less common than ignorance in many cases.
-
Educating women about ANC benefits is critical to improve attendance.
Ruling Out Other Options:
-
A) Visit is expensive
Possible, but many ANC programs are subsidized or free. -
C) Fear or anxiety
Less common compared to lack of knowledge.
Clinical Tip:
Effective community education programs can significantly improve ANC attendance and pregnancy outcomes.
Q.137. A patient tests HIV positive. You inform him and advise him to tell his wife. The patient requests you not to inform his wife. What should you do?
Correct Answer : C
Correct Answer:
C) Counsel the patient again to inform his wife; if he refuses, inform the wife as per legal and ethical guidelines
Explanation:
-
Patient confidentiality is paramount, but protecting the health of others (the partner) is also an ethical and legal obligation.
-
The physician should encourage the patient to disclose his status to the partner voluntarily.
-
If the patient refuses, the doctor may need to disclose to the partner to prevent harm, following local laws and protocols.
-
Reporting to health authorities is important for public health, but partner notification is a distinct obligation.
Ruling Out Other Options:
-
A) Do not tell the wife
Risks harm to partner and is unethical if patient refuses to disclose. -
B) Tell only health authorities
Does not protect the partner; partner notification is essential.
Clinical Tip:
Use partner notification protocols balancing confidentiality and duty to warn to prevent HIV transmission.
Q.138. An elderly female patient is diagnosed with cancer. Her sons ask you not to tell her about the diagnosis. What should you do?
Correct Answer : A
Correct Answer:
A) Tell the patient the truth about her diagnosis
Explanation:
-
Ethical principles and patient autonomy dictate that the patient has the right to know their diagnosis.
-
Withholding information violates the patient’s right to make informed decisions about their care.
-
Families’ wishes do not override patient autonomy unless the patient has expressed preference not to be informed.
-
Consulting health authorities can be helpful, but the primary duty is to be truthful to the patient.
Ruling Out Other Options:
-
B) Do not tell the patient
Violates autonomy and informed consent. -
C) Consult health authority
May be considered but does not replace the ethical obligation to inform the patient.
Clinical Tip:
Use sensitive, clear communication when delivering bad news; assess patient’s preferences but prioritize honesty.
Q.139. A patient with cystic fibrosis or celiac disease meets the doctor to understand how to manage the disease and its prognosis. What is this type of session called?
Correct Answer : B
Correct Answer:
B) Verbal communication
Explanation:
-
Verbal communication involves the exchange of information through spoken words, such as explaining diagnosis, management, and prognosis.
-
Summarization is a specific communication technique where key points are briefly restated to confirm understanding.
-
In this context, the session is primarily verbal communication as the doctor educates the patient.
Ruling Out Other Options:
-
A) Summarisation
Is a technique within communication, not the entire session.
Clinical Tip:
Use clear verbal communication complemented by written materials to improve patient understanding.
Q.140. A mother, who knows her child has diabetes and was administering insulin, suddenly stops giving insulin saying, "His pancreas is working now, and he has no disease anymore." This behavior is best described as:
Correct Answer : C
Correct Answer:
C) Denial
Explanation:
-
Denial is a psychological defense mechanism where a person refuses to accept the reality of a medical condition, leading to non-adherence.
-
This often happens in chronic illnesses where acceptance is difficult.
-
Anxiety may cause worry but not refusal of treatment.
-
Neglect involves disregard for patient care, but here the mother’s behavior is driven by denial.
Ruling Out Other Options:
-
A) Anxiety
Would present as worry or fear, not stopping treatment. -
B) Neglect
Implies intentional or unintentional failure to provide care, but this case is due to psychological denial.
Clinical Tip:
Identify denial early and provide psychological support and counseling to improve treatment adherence.
Q.141. A young child is newly diagnosed with asthma. The mother is also asthmatic, and the father is a chain smoker. What is the most important initial step in managing the child's condition?
Correct Answer : A
Correct Answer:
A) Stop smoking in the house
Explanation:
-
Environmental control, especially eliminating tobacco smoke exposure, is critical in managing pediatric asthma.
-
Secondhand smoke is a strong trigger for asthma exacerbations and worsens lung function.
-
While SABAs are important for symptom relief, addressing environmental triggers is a key preventive step.
Ruling Out Other Options:
-
B) Give short-acting beta-agonist
Necessary for acute symptom relief but does not address the root cause—smoke exposure.
Clinical Tip:
Educate families on avoiding tobacco smoke to reduce asthma attacks and improve long-term outcomes.
Q.142. Parents bring their child who is not vaccinated because they believe vaccines are harmful. What should you do?
Correct Answer : B
Correct Answer:
B) Explain to the parents about the myths and benefits of vaccination
Explanation:
-
Addressing vaccine hesitancy requires patient, clear education about the safety and importance of vaccines.
-
Correcting misinformation helps improve compliance and protects the child and community.
-
Consulting child protective services is reserved for cases of clear neglect or harm, not for initial refusal.
Ruling Out Other Options:
-
A) Consult child support (protective services)
Premature unless refusal leads to serious risk and after failed counseling.
Clinical Tip:
Use empathetic communication and evidence-based information to build trust and increase vaccination rates.
Q.143. A woman reports experiencing sexual harassment at work. Statistically, who is most commonly the perpetrator in such cases?
Correct Answer : C
Correct Answer:
C) Supervisor
Explanation:
-
Studies and workplace reports indicate that supervisors or those in positions of power are the most common perpetrators of sexual harassment.
-
Power dynamics often contribute to abuse, making supervisors more likely to be involved.
-
While co-workers can also be responsible, supervisory abuse often goes unreported due to fear of retaliation.
Ruling Out Other Options:
-
A) Employee – Too broad and nonspecific; doesn’t reflect hierarchy or control.
-
B) Co-worker – Possible but statistically less common than supervisors in reported cases.
Clinical Tip:
In medical or occupational settings, always follow institutional zero-tolerance policies and ensure proper reporting channels are in place for harassment cases.
Q.144. You arrive late for a clinic appointment, and the patient becomes visibly angry and starts shouting. What is the most appropriate initial response?
Correct Answer : B
Correct Answer:
B) Acknowledge the patient's feelings
Explanation:
-
In emotionally sensitive situations, empathy and active listening are the first steps toward de-escalation.
-
Acknowledging the patient's feelings helps build rapport and shows respect.
-
Explaining the reason for being late may be important later, but doing it first may seem defensive or dismissive of their frustration.
Ruling Out Other Option:
-
A) Explain to the patient why you were late
This might be interpreted as an excuse or avoidance of responsibility, especially if done before acknowledging their emotion.
Clinical Tip:
Use empathy before explanation in all emotionally charged encounters. Start with:
“I understand you're frustrated, and I truly apologize for the delay.”
Q.145. A patient comes to sign the consent form for a planned hysterectomy. What is the most appropriate step before she signs?
Correct Answer : A
Correct Answer:
A) Give her details about the surgery
Explanation:
-
Informed consent requires that the patient is fully informed about:
-
The nature of the procedure
-
Benefits
-
Risks and complications
-
Alternatives (including doing nothing)
-
-
Simply documenting consent without discussing details violates ethical and legal standards.
Ruling Out Other Option:
-
B) Make documentation clear
While important, clear documentation is secondary to ensuring the patient understands what she is consenting to. Consent is not valid if it is uninformed.
Clinical Tip:
Always prioritize communication before documentation. A well-informed patient is less likely to experience anxiety, confusion, or legal conflict.
Q.146. What is considered normal bereavement in terms of emotional response and duration?
Correct Answer : C
Correct Answer:
C) Not exceeding 12 months
Explanation:
-
Normal bereavement involves a range of emotional responses such as sadness, longing, and emotional pain following a loss.
-
While sadness is common, it is a symptom, not the duration of normal grief.
-
2-3 months may be typical for acute grief but normal bereavement can last up to a year. If symptoms persist longer, it may indicate complicated grief or depression.
-
Not exceeding 12 months is the generally accepted guideline for normal bereavement, as extended periods may require professional evaluation.
Ruling Out Other Options:
-
A) Sadness
Sadness is a component, but bereavement also involves other emotional responses, not just sadness. -
B) 2-3 months
This is the typical period for acute grief, but normal bereavement can last longer (up to a year).
Clinical Tip:
Grief counseling or support can help individuals cope with the complex emotions of bereavement, particularly if it continues beyond 12 months.
Q.147. A 28-year-old woman with intermittent asthma uses albuterol (SABA) as needed. Over the past month, she has had daytime symptoms more than twice per week and night awakenings once a week. Her physician recommends Step 2 therapy, but she is reluctant to start daily inhaled corticosteroids (ICS) due to fear of side effects. What is the most appropriate next step in management?
Correct Answer : C
Correct Answer: C. Educate the patient on the correct use and benefits of inhaled corticosteroids
Explanation:
This patient meets criteria for Step 2 asthma management per GINA guidelines:
-
Symptoms >2x/week
-
Nighttime awakenings
-
Need for step-up therapy (low-dose ICS)
However, the patient refuses escalation due to concerns over ICS side effects.
The best clinical step is to educate the patient:
-
Explain how inhaled corticosteroids reduce airway inflammation, prevent exacerbations, and do not cause systemic side effects when used correctly.
-
Emphasize proper inhaler technique and low systemic absorption.
Only after education fails, alternative controller options (e.g., LTRA) might be considered.
Ruling Out Other Options:
-
A. Increase dose of SABA
-
Incorrect. Overuse of SABA without a controller increases mortality risk.
-
It does not address underlying inflammation and is not a step-up therapy.
-
-
B. Add LABA
-
LABA should never be used without an ICS due to increased risk of asthma-related death.
-
Only appropriate in Step 3 and above, always combined with ICS.
-
-
D. Prescribe LTRA
-
Can be considered if patient continues to refuse ICS, but it is less effective.
-
First-line remains ICS; this is a second-line alternative, not the immediate next step.
-
Clinical Tip:
Patient education can be therapeutic.
Before escalating or changing medication, ensure the patient:
-
Understands the role of ICS (controller vs. reliever)
-
Is reassured about safety and minimal systemic absorption
-
Knows proper inhaler technique to maximize delivery and minimize side effects
Non-adherence often stems from fear or misinformation—address that first.
Q.148. An 82-year-old man with hypertension and type 2 diabetes is found to have poorly controlled blood pressure and glucose levels on recent follow-up. On questioning, he admits he often forgets to take his medications. He lives alone and has mild cognitive decline. What is the most appropriate next step to improve his medication adherence?
Correct Answer : B
Correct Answer: B. Involve his family members in managing his medication regimen
Explanation:
In elderly patients—especially those with cognitive decline or functional impairment—noncompliance is often due to forgetfulness, confusion, or lack of support. The most effective and immediate intervention is to:
-
Engage family or caregivers to assist with reminders, pill organizers, or even medication administration.
-
This is especially crucial in patients living alone or with mild cognitive issues.
Simply warning the patient is ineffective and may reduce trust. Adjusting medications or simplifying dosing helps only after support systems are addressed.
Ruling Out Other Options:
-
A. Warn him about dangers
-
Ineffective and possibly counterproductive in elderly with cognitive impairment. May lead to fear, guilt, or withdrawal from care.
-
-
C. Prescribe more medications
-
Increases complexity and risk of adverse effects. Does not address the root cause, which is non-adherence.
-
-
D. Switch to once-daily dosing
-
May help in mild cases, but doesn’t resolve forgetfulness or ensure compliance in someone with cognitive issues.
-
Clinical Tip:
Nonadherence in elderly is rarely intentional.
Always assess for:
-
Cognitive status
-
Social support
-
Barriers to adherence (e.g., vision, manual dexterity, understanding)
Involving family or caregivers is often more effective than changing the regimen.
Q.149. A 13-year-old child presents with bilateral leg burns. The father reports the child accidentally walked on a hot surface. On physical examination, you notice another older lesion that appears unrelated and due to neglect or carelessness. The history is inconsistent with the pattern of injuries. What is the most appropriate next step in management according to child protection protocols?
Correct Answer : A
Correct Answer: A. Document findings clearly and report to child protective services
Explanation:
This scenario raises high suspicion of non-accidental injury (child abuse):
-
Bilateral, patterned, or symmetric burns (especially on legs or buttocks) are classic signs.
-
An additional lesion from a different time frame suggests recurrent injury, possibly due to neglect or abuse.
In such cases, healthcare providers are legally and ethically obligated to:
-
Document all findings thoroughly
-
Report the case to child protection authorities immediately
-
Ensure the child’s safety
Delaying action or follow-up may put the child at continued risk.
Ruling Out Other Options:
-
B. Schedule follow-up
-
Inappropriate. Delays protection and increases risk of further harm. Suspected abuse requires immediate action, not observation.
-
-
C. Ask for more details and accept explanation
-
Dangerous. Inconsistent history with physical findings should never be dismissed based on verbal reassurance alone.
-
-
D. Refer to dermatology
-
Misses the critical issue. This is not a dermatological case—it’s a safeguarding issue. Delaying reporting may be legally negligent.
-
Clinical Tip:
In suspected child abuse, your duty is to the child, not the parent.
-
You are a mandatory reporter.
-
Always document clearly (location, shape, size of injuries).
-
Report to child protection services promptly, even if you're not certain—it is not your job to confirm abuse, only to suspect and report.
Q.150. A 6-year-old girl is brought to your clinic for a routine check-up. On examination, you note multiple café-au-lait spots (>6), each measuring more than 5 mm. The child is otherwise healthy. The mother says, “These birthmarks run in our family.” What is the most appropriate next step?
Correct Answer : C
Correct Answer: C. Provide counseling about Neurofibromatosis Type 1 (NF1)
Explanation:
Café-au-lait macules can be benign, but when ≥6 lesions >5 mm (prepubertal) or >15 mm (postpubertal) are found, especially in a child, they raise concern for neurofibromatosis type 1 (NF1).
-
NF1 is a genetic disorder with autosomal dominant inheritance.
-
Diagnosis is clinical and may precede the appearance of other signs such as neurofibromas, axillary freckling, Lisch nodules, or optic gliomas.
-
Initial step is clinical counseling about the possibility of NF1, monitoring, and genetic evaluation if needed.
Thus, counseling is the first step, and referrals can follow based on findings.
Ruling Out Other Options:
-
A. General genetic education
-
Too vague and nonspecific. It doesn’t directly address the clinical suspicion or inform the family properly.
-
-
B. Genetic health education about NF1
-
Similar to A in scope and premature. Education is important but only after clinical suspicion is explained and appropriate counseling is provided.
-
-
D. Reassure and discharge
-
Inappropriate. While the family may have a history of birthmarks, café-au-lait spots can indicate a serious genetic syndrome, and evaluation is warranted.
-
Clinical Tip:
Multiple café-au-lait spots are not always benign.
-
≥6 spots meeting the size criteria is a diagnostic criterion for NF1.
-
Early diagnosis helps monitor for complications: learning disabilities, vision loss, scoliosis, and tumors.
-
Clinical counseling comes first. Family education and genetic testing follow.
Q.151. An ICU physiotherapist notices redness over the back of an intubated patient during mobilization and informs the nurse, suspecting an early pressure ulcer. The nurse forgets to communicate this finding to the resident. The resident remains unaware, and no further action is taken. What is the primary issue in this scenario?
Correct Answer : C
Correct Answer: C. This is a case of poor communication
Explanation:
This scenario exemplifies a breakdown in interprofessional communication, which is a common and preventable cause of adverse patient outcomes in ICU settings.
Key issues:
-
The physiotherapist relayed important clinical information.
-
The nurse failed to pass on this information, breaking the communication chain.
-
No system was in place to ensure escalation or documentation.
The root cause here is system-level communication failure, not just individual error. Blaming or replacing the nurse may not resolve the underlying structural issue.
Ruling Out Other Options:
-
A. The nurse is responsible
-
Partially true, but this places blame on an individual, which is not in line with modern patient safety practices that emphasize system improvement.
-
-
B. The nurse should be replaced
-
Not justified. This is a harsh and reactive response to a communication lapse, and doesn’t improve the system or prevent recurrence.
-
-
D. The physiotherapist should have documented it
-
While documentation is ideal, primary responsibility for medical action lies with the clinical team. The failure is more in communication relay and coordination, not lack of documentation alone.
-
Clinical Tip:
Use structured communication tools like SBAR (Situation, Background, Assessment, Recommendation) in ICU or inpatient settings.
They:
-
Reduce communication gaps
-
Ensure consistent information transfer
-
Improve teamwork and patient safety
Q.152. A 55-year-old man with diabetes is diagnosed with advanced Charcot foot (neuropathic arthropathy). The treating physician recommends below-knee amputation due to deformity and risk of chronic infection. The patient, fully oriented and competent, refuses the procedure, stating he wants to try alternative options. What is the most appropriate next step?
Correct Answer : B
Correct Answer: B. Respect the patient's decision and document the refusal
Explanation:
This case is about patient autonomy, a core principle in medical ethics.
-
A competent adult patient has the legal and ethical right to refuse any treatment, even if that decision may lead to worse outcomes or death.
-
As long as the patient is mentally sound, the physician must:
-
Respect the decision
-
Provide appropriate counseling
-
Document the discussion and refusal clearly
-
Forcing treatment or acting against the patient's will violates ethical and legal standards.
Ruling Out Other Options:
-
A. Refer to another doctor for second opinion
-
Not necessary unless the patient requests it. Respecting autonomy is still required.
-
-
C. Ask the patient to sign for discharge
-
Premature and inappropriate. The patient is refusing one specific intervention, not all care. He can still receive wound care, offloading, or orthotic management.
-
-
D. Proceed with surgery in best interest of patient
-
This violates patient autonomy and could be considered battery if performed without consent.
-
Clinical Tip:
Always balance beneficence (doing good) with autonomy.
Even if the recommended treatment is lifesaving, a competent patient can refuse it.
Key steps:
-
Provide detailed counseling
-
Ensure understanding of consequences
-
Document clearly
-
Offer alternative management within the patient’s values
Q.153. You are a resident physician working in a hospital. A nurse arrives inappropriately dressed in a transparent uniform and engages in seductive, unprofessional behavior during a conversation. What is the most appropriate next step?
Correct Answer : B
Correct Answer: B. Report the behavior to the medical director or supervisor
Explanation:
This situation involves unprofessional conduct that could disrupt the workplace environment, violate institutional codes of ethics, and may lead to harassment or boundary issues.
-
It is not the physician’s role to confront or discipline staff directly in this context.
-
The correct course of action is to report the inappropriate behavior to hospital administration, typically the Medical Director or Nurse Supervisor, to handle the issue through proper channels.
This maintains professional boundaries and ensures the situation is addressed safely and legally.
Ruling Out Other Options:
-
A. Immediately walk her out and replace her
-
Inappropriate and confrontational. This is not your administrative role, and doing so could escalate the situation or lead to legal consequences.
-
-
C. Leave the office immediately
-
Avoiding the situation does not resolve the professional misconduct. The issue still needs to be formally reported.
-
-
D. Ignore the behavior to avoid conflict (not listed but important)
-
Ignoring such behavior may allow a toxic or unsafe workplace culture to continue. Physicians have a duty to maintain professional standards.
-
Clinical Tip:
When faced with unprofessional conduct in the clinical setting:
-
Stay calm and non-confrontational
-
Avoid personal or disciplinary action
-
Use the chain of command: report to your supervisor, medical director, or HR
-
Always document the incident objectively if needed later
This ensures a safe, respectful work environment and protects both patients and staff.
Q.154. You are conducting a HEADSSS assessment (Home, Education, Activities, Drugs, Sexuality, Suicide, Safety) for a 15-year-old adolescent who has been referred for behavioral concerns. To ensure accurate and open disclosure, how should this interview be conducted?
Correct Answer : B
Correct Answer: B. Alone with the adolescent
Explanation:
The HEADSSS psychosocial interview is designed to explore sensitive areas of an adolescent's life, including sexual behavior, substance use, mental health, and safety concerns.
-
Adolescents are much more likely to disclose sensitive information when interviewed privately without parents present.
-
The standard practice is to begin with both (to establish rapport and consent), but then ask parents to leave for the focused HEADSSS interview.
-
Confidentiality should be explained beforehand, with clear exceptions (e.g., risk of harm to self or others).
Ruling Out Other Options:
-
A. In the presence of parents throughout
-
Incorrect. This limits the adolescent’s willingness to share sensitive issues like sexual activity, abuse, or drug use.
-
-
C. With both adolescent and parents present together
-
This is only useful during initial introductions, not during the core assessment.
-
-
D. Let the adolescent decide if parents can be present
-
While adolescent comfort matters, the clinician has a professional duty to conduct private screening for accurate assessment.
-
Clinical Tip:
Always build rapport first, then explain confidentiality:
“Everything you tell me is confidential unless you're in danger or someone else is at risk.”
This opens the door to trust, and most adolescents will open up significantly more once parents leave the room.
Q.155. A Type 2 diabetic patient has a clinic appointment scheduled for 11:00 AM. The doctor arrives at 12:30 PM due to a serious emergency in the hospital. The patient is visibly angry and upset about the delay. As the doctor, what is the best immediate response?
Correct Answer : B
Correct Answer: B. Explore the patient’s anger and listen to their concerns
Explanation:
-
Simply apologizing is polite but may not fully address the patient’s frustration.
-
The best approach is to actively explore and acknowledge the patient’s feelings by asking open-ended questions like:
“I can see you’re upset about the wait. Can you tell me what’s on your mind?” -
This shows empathy, improves rapport, and can help defuse tension.
-
Following exploration, a clear explanation and apology can be offered.
-
Ignoring emotions or only explaining the delay without acknowledgment may increase patient dissatisfaction.
Ruling Out Other Options:
-
A. Apologize and expect understanding
-
Politeness alone may seem dismissive without engaging the patient’s feelings.
-
-
C. Explain why you were late without addressing emotions
-
Fails to validate patient’s emotional experience, reducing trust.
-
-
D. Ignore anger and proceed with consultation
-
Likely to worsen patient dissatisfaction and decrease adherence.
-
Clinical Tip:
Always address the patient’s emotions first.
Active listening and empathy are key to building trust, especially when patients feel frustrated or neglected. This approach reduces complaints and improves clinical outcomes.
Q.156. A patient whom you treated offers you an expensive watch as a gift to show gratitude. What is the most appropriate action?
Correct Answer : B
Correct Answer: B. Politely refuse the gift
Explanation:
-
Accepting expensive gifts from patients can compromise professional boundaries and may create conflicts of interest or perceived favoritism.
-
The best practice is to politely refuse such gifts while appreciating the patient’s gratitude.
-
Small, symbolic gifts (like handmade cards or baked goods) may sometimes be acceptable depending on institutional policy.
-
If refusal is difficult, explaining hospital policy or ethics guidelines can help.
-
Documenting the interaction may be advisable if there is concern about persistence.
Ruling Out Other Options:
-
A. Accept the gift and ask not to do it again
-
Accepting valuable gifts can still create ethical dilemmas and affect trust.
-
-
C. Accept the gift
-
Unethical and potentially leads to conflicts or expectations.
-
-
D. Donate the gift to charity
-
Although well-intentioned, this does not avoid the ethical concern of accepting the gift initially.
-
Always maintain clear professional boundaries.
If patients want to express gratitude, suggest non-material ways such as thank-you notes or positive feedback to hospital administration.
Q.157. A physician is invited to present at a hypertension conference. A pharmaceutical company that produces a new antihypertensive drug offers to sponsor his trip. What is the most ethical and appropriate action?
Correct Answer : C
Correct Answer: C. Politely refuse the sponsorship offer
Explanation:
-
Accepting sponsorship from a company that may benefit financially from the presentation risks conflicts of interest and may bias the doctor’s content or perceived objectivity.
-
The best ethical practice is to decline such sponsorships to maintain professional integrity and public trust.
-
Disclosure (Option A) is important if sponsorship exists, but avoiding sponsorship altogether is preferred to avoid any real or perceived bias.
-
Simply disclosing the sponsorship (Option A) may not be sufficient to prevent bias.
-
Avoiding mention of the drug (Option B) does not address the conflict of interest.
-
Accepting sponsorship but preparing the presentation independently (Option D) is often discouraged due to perceived influence.
Ruling Out Other Options:
-
A. Disclose sponsorship
-
Disclosure is necessary if sponsorship is accepted but does not eliminate conflict.
-
-
B. Avoid mentioning the new drug
-
Does not prevent conflict and may limit educational content.
-
-
D. Accept sponsorship but prepare independently
-
Perceived conflict remains, risking credibility.
-
Clinical Tip:
Maintain transparency and independence.
Avoiding financial ties to industry representatives helps preserve trust and prevents undue influence on medical education or practice.
Q.158. A surgeon diagnoses an inguinal hernia in a child and recommends urgent surgical repair. The child’s mother refuses consent for surgery. The hernia is not strangulated or an emergency. What is the most appropriate next step?
Correct Answer : D
Correct Answer: D. Explain the diagnosis, risks, benefits, and consequences to the mother
Explanation:
-
In cases where parents refuse consent for non-emergency surgery, the physician must first engage in detailed discussion with the parent.
-
Explain clearly the nature of the condition, potential risks of delaying surgery (such as incarceration or strangulation), and benefits of timely repair.
-
This is part of the informed consent process aimed at helping the parent make an informed decision.
-
If refusal persists and the child is at significant risk, legal or child protection pathways may be pursued, but only after thorough counseling.
-
Immediate surgery without consent is only justified in true emergencies threatening life or limb.
Ruling Out Other Options:
-
A. Do surgery despite refusal
-
Unethical and illegal unless the child is in immediate life-threatening danger.
-
-
B. Inform hospital police immediately
-
Premature without trying to resolve through communication; police involvement is last resort in cases of neglect/endangerment.
-
-
C. Do not perform surgery and simply respect refusal
-
Passive approach; must ensure parents understand the risks and consequences first.
-
Clinical Tip:
Effective communication and empathy are critical in parental refusal cases.
Use simple language, address fears or misconceptions, and document all discussions carefully. Legal intervention is a last step if child safety is at risk.
Q.159. A female patient presents to the ER with signs and symptoms of appendicitis. An appendectomy is performed, but intraoperatively, the surgeon finds a grossly normal appendix and no clear alternate diagnosis. What is the most appropriate next step?
Correct Answer : A
Correct Answer: A. Inform the patient about the findings and explain that this can occur and there were no complications
Explanation:
-
Transparency and honesty are essential components of medical ethics.
-
When a procedure is performed based on clinical suspicion (e.g., appendicitis), and intraoperative findings are unexpected (e.g., normal appendix), the surgeon must disclose this to the patient.
-
This maintains trust, upholds ethical standards, and aligns with the principle of informed care.
-
It is also important to reassure the patient that the surgery was indicated based on clinical signs, and such outcomes are not uncommon in surgical practice.
Ruling Out Other Options:
-
B. Do not tell the patient
-
Unethical. Withholding information undermines patient autonomy and can lead to loss of trust.
-
-
C. Write an internal report only
-
Documentation is essential, but it does not replace the duty to disclose findings to the patient.
-
-
D. Blame imaging
-
Inappropriate and unprofessional. Diagnostic uncertainty exists; blaming others is defensive and unethical.
-
Clinical Tip:
In non-therapeutic laparotomies (when expected pathology is not found), always:
-
Explain findings clearly and calmly to the patient.
-
Reassure them that surgery was based on appropriate judgment.
-
Document intraoperative findings and communication thoroughly.
Q.160. A patient with a known history of inguinal hernia presents to the clinic now complaining of pain. After clinical evaluation, you determine that surgery is not currently indicated. The patient becomes upset and states he wants to file a complaint against you for not performing the surgery. What is the most appropriate next step?
Correct Answer : C
Correct Answer: C. Refer the patient to another qualified surgeon for second opinion
Explanation:
-
In situations where a patient disagrees with your medical judgment, especially involving invasive procedures like surgery, it is appropriate and ethical to offer a second opinion.
-
This respects the patient’s autonomy and provides reassurance that they are receiving proper care.
-
It also protects the physician legally and ethically by demonstrating openness and professionalism.
-
You should also remain calm, avoid confrontation, and document the conversation.
Ruling Out Other Options:
-
A. Perform the surgery to satisfy the patient
-
Unethical and unsafe to perform surgery without medical indication. It violates the principle of non-maleficence.
-
-
B. Call hospital security
-
Premature and inappropriate unless the patient becomes physically threatening or violent.
-
-
D. Ignore the complaint
-
Dismissive approach; escalates patient dissatisfaction and does not respect the patient's right to seek further evaluation.
-
Clinical Tip:
When in doubt or under pressure, refer.
Referring for a second opinion maintains professional boundaries and ensures shared responsibility, especially in contentious or uncertain situations.
Q.161. What is the most effective way to inform the public about the dangers of poisoning and promote prevention?
Correct Answer : D
Correct Answer: D. Conduct structured awareness campaigns explaining the dangers and prevention strategies
Explanation:
-
Public health awareness campaigns are the most comprehensive and impactful method to educate society about poisoning risks.
-
They include a combination of media, community engagement, school programs, and printed materials.
-
Campaigns allow repetitive messaging, broad reach, and sustained behavior change across different populations.
-
Poison awareness is especially critical in preventing accidental ingestions in children, chemical misuse, and carbon monoxide poisoning.
Ruling Out Other Options:
-
A. Use media platforms
-
Helpful tool within a campaign, but media alone is not enough without structure or reinforcement.
-
-
B. Arrange a one-time camp
-
One-time events lack continuity; may not effectively change long-term behavior or reach all audiences.
-
-
C. Tell fathers and mothers personally
-
Limited reach. Individual education is important, but not scalable for community-level awareness.
-
Clinical Tip:
Public health messaging should be repeated, community-focused, and culturally appropriate.
Use poison control center numbers, pictorial posters, and real-life scenarios to create lasting awareness, especially in low-literacy populations.
Q.162. During a laparoscopic cholecystectomy, a common bile duct (CBD) injury occurs but is immediately recognized and effectively repaired intraoperatively. The patient is stable postoperatively. What is the most appropriate next step?
Correct Answer : A
Correct Answer: A. Disclose the complication to the patient and reassure them
Explanation:
-
Ethical medical practice mandates full disclosure of intraoperative complications, even if the complication is recognized and successfully managed.
-
Transparency builds trust, respects patient autonomy, and reduces medico-legal risk.
-
Reassurance is important, but must come after an honest explanation of what occurred and how it was resolved.
-
The patient has a right to know all relevant details about their surgery.
Ruling Out Other Options:
-
B. Reassure but don't tell
-
Unethical. Concealing surgical complications violates patient rights and informed care.
-
-
C. Inform committee only
-
Internal reporting is important, but does not substitute patient disclosure.
-
-
D. Wait for symptoms to appear
-
Inappropriate. Waiting until symptoms appear before disclosure is reactive and unethical.
-
Clinical Tip:
When discussing a surgical complication:
-
Use clear, non-technical language.
-
Emphasize that the situation was promptly recognized and managed.
-
Offer a plan for monitoring and be open to patient questions.
Q.163. A physician tells a patient: “I will be discussing with you the diagnosis of your child’s disease and the expected outcomes. After that, I will answer any questions you may have.” This statement reflects which stage of effective communication?
Correct Answer : B
Correct Answer: B. Setting the agenda
Explanation:
-
The physician is organizing the flow of the consultation by outlining what will be discussed (diagnosis, outcomes, then questions).
-
This is a classic example of setting the agenda, which helps manage time, improves understanding, and ensures both patient and physician are aligned in expectations.
-
It’s a key step in structured, patient-centered communication, especially in emotionally sensitive discussions like childhood illness.
Ruling Out Other Options:
-
A. Assessing understanding
-
Happens after information has been shared, typically by checking what the patient has understood.
-
-
C. Creating a comfortable setting
-
Refers to preparing the environment (e.g., privacy, seating), not outlining the discussion plan.
-
-
D. Probing
-
Involves asking in-depth questions to explore feelings or hidden concerns, not organizing the discussion.
-
Clinical Tip:
Setting an agenda at the beginning of a consultation:
-
Shows respect for the patient’s time and emotions.
-
Reduces anxiety by clarifying the process.
-
Makes the encounter more productive and structured, especially in pediatric or sensitive scenarios.
Q.164. A 1-year-old child is brought to the Emergency Department cyanotic and irritable, with a history from the mother that he had been crying continuously for 1 hour. She reports that the child turned blue during crying, but there is no fever, vomiting, or trauma. He is alert but agitated. What is the most appropriate initial step in management?
Correct Answer : A
Correct Answer: A. Calm the child and reassure the mother
Explanation:
-
This clinical presentation is most consistent with a breath-holding spell, particularly the cyanotic type, which is benign and self-limiting.
-
Breath-holding spells typically occur in infants and toddlers, often triggered by emotional stimuli like anger, frustration, or pain.
-
Children may cry intensely, then suddenly stop breathing, turn cyanotic, and even briefly lose consciousness — but they recover spontaneously within seconds to a minute.
-
Management is supportive: calm the child, reassure the parents, and educate them on the benign nature.
Ruling Out Other Options:
-
B. Surgical intervention
-
No signs of a surgical emergency like intussusception, incarcerated hernia, or trauma.
-
-
C. High-flow oxygen and intubation
-
Not needed unless the child is apneic and unresponsive — here, the child is agitated but alert.
-
-
D. Emergency CT brain
-
Not warranted without signs of neurological trauma, seizure, or altered mental status.
-
Clinical Tip:
Cyanotic breath-holding spells are common between 6 months and 6 years.
Reassure caregivers, but screen for iron deficiency anemia, as it’s often associated and treatment reduces frequency.
Q.165. A 37-week gestation woman, G2P1 with one prior low-transverse cesarean section, is diagnosed with intrauterine fetal demise (IUFD) on ultrasound. She is clinically stable, afebrile, and all maternal investigations are within normal limits. What is the most appropriate next step?
Correct Answer : C
Correct Answer: C. Explain all management options—including expectant management—and allow her to decide timing of delivery
Explanation:
-
In term IUFD with a prior cesarean, the main goals are to minimize maternal risk and respect patient autonomy.
-
Vaginal delivery is usually preferred even after prior low-transverse cesarean, but induction carries a small risk of uterine rupture.
-
Expectant management (waiting up to 1–2 weeks) is an acceptable option if the patient is stable and understands:
-
Risks of coagulopathy increase after 2–3 weeks (monitor coagulation profiles every 1–2 weeks).
-
Potential for uterine infection or psychological distress.
-
-
The physician’s duty is to counsel on risks and benefits of:
-
Immediate induction (shorter morbidity)
-
Expectant management (less intervention but increased monitoring)
-
Repeat cesarean (more morbidity, generally reserved for other indications)
-
-
Final decision rests with the informed patient.
Ruling Out Other Options:
-
A. Repeat cesarean delivery
-
Unnecessary surgical morbidity when vaginal birth is safe and preferred.
-
-
B. Immediate induction with oxytocin
-
Reasonable if the patient opts for it, but should not be imposed without discussion of risks and alternatives.
-
-
D. Immediate D&E under general anesthesia
-
Not appropriate for a 37-week fetus; D&E is for first- and second-trimester loss.
-
Clinical Tip:
Monitor for coagulopathy in IUFD: Check fibrinogen and D-dimer weekly if expectant management extends beyond 7 days. Engage mental health support early, as psychological distress is common with delayed delivery.
Q.166. A 72-year-old man presents with exertional dyspnea and chest pain that has worsened over the last 6 months. He also reports occasional episodes of dizziness. On examination, he has a harsh, crescendo-decrescendo systolic murmur best heard at the right upper sternal border, radiating to the carotids. His carotid pulse is weak and delayed. Echocardiography reveals a thickened and calcified aortic valve with reduced mobility. Which of the following is the most likely diagnosis?
Correct Answer : B
Correct Answer: B. Aortic stenosis
Explanation
This patient presents with the classic triad of aortic stenosis:
-
Angina
-
Syncope
-
Dyspnea on exertion (heart failure symptoms)
Physical exam findings:
-
Harsh, crescendo-decrescendo systolic murmur at the right upper sternal border, radiating to the carotids
-
Pulsus parvus et tardus (weak and delayed carotid pulse)
These are hallmark features of severe calcific aortic stenosis, common in elderly patients due to age-related valve degeneration.
Ruled-Out Options
-
A. Aortic regurgitation
-
Presents with a diastolic decrescendo murmur at the left sternal border.
-
Associated with bounding pulses and wide pulse pressure, not delayed carotid upstroke.
-
-
C. Hypertrophic obstructive cardiomyopathy (HOCM)
-
Also causes a systolic murmur, but it is best heard at the left lower sternal border.
-
Murmur increases with Valsalva and decreases with squatting, and usually occurs in younger patients.
-
-
D. Mitral valve prolapse
-
Causes a mid-systolic click followed by a late systolic murmur, best heard at the apex.
-
Commonly seen in young women and associated with connective tissue disorders.
-
Clinical Tip
To distinguish aortic stenosis from other systolic murmurs, remember:
"SAD symptoms in the old with a murmur that radiates to the carotids = Aortic Stenosis"
-
S = Syncope
-
A = Angina
-
D = Dyspnea
Also, feel the carotid pulse—slow rising (parvus et tardus) is very specific.
Q.167. A 26-year-old woman brings her 9-month-old infant to the clinic. The baby appears underweight and shows signs of malnutrition. On further questioning, you learn that the mother is from a low socioeconomic background and is struggling to afford adequate nutrition. She seems cooperative but overwhelmed and tearful. You suspect that social factors are contributing significantly to the child's condition. What is the most appropriate next step in management?
Correct Answer : A
Correct Answer: A. Ask close-ended questions and refer her to a social worker
Explanation
This scenario reflects a socially vulnerable mother with a malnourished child, likely due to poverty and lack of resources rather than willful neglect. The best initial step is to:
-
Use close-ended, nonjudgmental questions to quickly gather specific, actionable information (e.g., "Do you have access to regular meals?" "Do you receive any financial aid or food support?")
-
Then, refer her to a social worker, who can help with nutritional support programs, housing, food security, and welfare services.
This approach ensures holistic care by addressing social determinants of health without making premature assumptions or alienating the patient.
Ruled-Out Options
-
B. Psychiatric consultation
-
Not indicated unless there is evidence of mental illness (e.g., postpartum depression, psychosis). Being overwhelmed is expected in her situation and doesn’t warrant psychiatry yet.
-
-
C. Inpatient nutritional rehabilitation
-
Appropriate only if the infant is severely malnourished with complications like dehydration, electrolyte imbalance, or infection. Based on the vignette, outpatient support with social services is the first step.
-
-
D. Report to child protective services
-
Reporting is only appropriate if there's clear evidence of abuse or neglect. In this case, the situation is driven by poverty, not intentional harm, so this is premature and potentially damaging.
-
Clinical Tip
In cases of pediatric malnutrition from low-income families, social intervention is key.
Always assess for underlying social factors before medicalizing or criminalizing the situation. A nonjudgmental, supportive approach builds trust and leads to better outcomes.
Q.168. A 62-year-old woman presents with postmenopausal bleeding for the past 2 months. She underwent a transvaginal ultrasound, which revealed an endometrial thickness of 15 mm. An endometrial biopsy was performed and shows complex atypical hyperplasia. You decide to proceed with a hysterectomy as the next step. What is the most appropriate immediate action before scheduling surgery?
Correct Answer : D
Correct Answer: D. Obtain written informed consent
Explanation
The patient is postmenopausal with abnormal uterine bleeding and endometrial thickening (>4 mm). Biopsy reveals complex atypical hyperplasia, which carries a high risk of progression to endometrial carcinoma.
The definitive treatment is total hysterectomy, often with bilateral salpingo-oophorectomy.
Before proceeding with surgery, the first and most appropriate step is to:
-
Obtain written informed consent, which includes discussing:
-
Indication for surgery
-
Risks and benefits
-
Alternatives
-
Potential complications
-
Verbal consent is not sufficient for major surgery. Hospital admission and anesthesia consult are done after proper consent.
Ruled-Out Options
-
A. Call the anesthetist
-
Important step before surgery but comes after informed consent is obtained.
-
-
B. Take verbal consent
-
Not legally or ethically adequate for a surgical procedure. Surgery requires written informed consent.
-
-
C. Admit the patient to the hospital
-
Admission happens after surgical planning, not before proper documentation and consent.
-
Clinical Tip
In postmenopausal women, endometrial thickness >4 mm is abnormal.
If biopsy reveals complex atypical hyperplasia, prompt hysterectomy is indicated.
Always document written informed consent before proceeding to major procedures.
Q.169. A 35-year-old woman with a 10-year history of infertility and irregular menstrual cycles presents with vaginal bleeding. She had recently missed a period and tested positive on a home pregnancy test. An ultrasound confirms an incomplete miscarriage. She is visibly distressed and her husband appears emotionally affected as well. What is the most appropriate immediate approach after confirming the diagnosis?
Correct Answer : C
Correct Answer: C. Show empathy and console the couple’s emotions
Explanation
This patient has experienced emotional trauma due to pregnancy loss after long-standing infertility, a deeply distressing situation for any couple.
After confirming the incomplete miscarriage, the most appropriate immediate step is not clinical intervention, but addressing the emotional response through:
-
Empathy: Acknowledge their pain ("I can see this is very difficult for you.")
-
Consoling: Offer emotional support in a non-judgmental, compassionate manner
-
Include both partners in the discussion, validating their feelings
While treatment for incomplete miscarriage (e.g., misoprostol or D&C) is necessary, addressing emotional needs comes first.
Ruled-Out Options
-
A. Show empathy and sympathy
-
"Sympathy" can sometimes feel patronizing or superficial. Consoling is more appropriate as it involves active emotional support.
-
-
B. Empathy only
-
While important, just empathy without emotional support is incomplete. You must follow up with consolation and reassurance.
-
-
D. Begin medical evacuation without delay
-
Rushing into clinical action without acknowledging emotional distress can harm the therapeutic relationship. This should come after emotional care.
-
Clinical Tip
In cases of miscarriage, especially following infertility, always prioritize emotional support before clinical steps.
Use the SPIKES protocol for breaking bad news:
S – Set up the interview
P – Perception (assess what they know)
I – Invitation (permission to give news)
K – Knowledge (share information sensitively)
E – Empathy
S – Strategy and summary
Q.170. An elderly patient with terminal illness has a Do Not Resuscitate (DNR) order documented in the medical record. However, the patient’s family refuses to accept the DNR and accuses the doctor of arrogance during a meeting. What is the most appropriate approach for the physician in the family meeting?
Correct Answer : B
Correct Answer: B. Inform the family about the patient's prognosis and explain the DNR
Explanation
In cases of family refusal or conflict over a DNR order, the physician’s role is to:
-
Inform the family clearly and compassionately about the patient’s prognosis, medical condition, and the purpose and implications of the DNR order.
-
Use effective communication, including empathy and listening to concerns, to foster understanding and shared decision-making.
-
Avoid defensiveness or confrontation, which can escalate mistrust.
Justifying or defending the decision aggressively can worsen the conflict and alienate the family.
Ruled-Out Options
-
A. Justify the DNR decision firmly to the family
-
This approach risks appearing arrogant or dismissive, potentially escalating conflict. The focus should be on clear, empathetic communication rather than justification or debate.
-
-
C. Avoid the meeting to prevent conflict
-
Avoidance undermines trust and leaves the family uninformed.
-
-
D. Override the family and proceed with full resuscitation
-
Resuscitation against documented DNR is unethical and legally questionable.
-
Clinical Tip
When discussing DNR orders with families, focus on:
Clear information about prognosis
Respectful listening of family concerns
Avoiding defensive or confrontational language
Utilizing palliative care teams or ethics consultations if needed
Q.171. A 28-year-old woman at 10 weeks of pregnancy is diagnosed with ovarian cancer. She expresses her wish to terminate the pregnancy to begin cancer treatment promptly. What is the most appropriate next step in managing this situation?
Correct Answer : A
Correct Answer: A. Inform the hospital ethics committee
Explanation
Cancer diagnosis during early pregnancy is a complex situation involving medical, ethical, and legal considerations.
-
The hospital ethics committee plays a crucial role in guiding complex decisions like pregnancy termination in cases of maternal malignancy.
-
They provide a multidisciplinary review balancing patient autonomy, fetal viability, legal framework, and medical urgency.
-
This is especially important in regions with specific legal restrictions on abortion (e.g., Saudi Arabia).
-
Consulting the ethics committee ensures transparent, ethically sound, and legally compliant care.
Ruled-Out Options
-
B. Search the website of Saudi law regarding abortion
-
Knowing the law is important but decision-making should be multidisciplinary and institutional, not based solely on personal internet search.
-
-
C. Ask your colleague for advice
-
Informal advice is insufficient for such serious decisions needing formal ethical guidance.
-
-
D. Proceed with abortion immediately after consent
-
Immediate action without institutional ethical approval and legal consideration can lead to complications and liability.
-
Clinical Tip
In oncology during pregnancy, always involve a multidisciplinary team, including the ethics committee, to balance maternal treatment urgency and fetal considerations under the legal framework.
Q.172. A patient undergoes a tonsillectomy. During surgery, the surgeon encounters unexpected difficulty controlling bleeding, which prolongs the operation. The procedure is completed without complications. The next day, the patient asks why the surgery took longer than expected. What is the most appropriate response by the surgeon?
Correct Answer : B
Correct Answer: B. Tell the patient the truth and reassure them that there were no complications
Explanation
Open, honest communication after surgery is critical for maintaining patient trust and professional integrity.
-
The patient has a right to know about unexpected events during surgery.
-
Disclosing the difficulty and reassuring that the issue was managed without complications helps reduce anxiety.
-
Avoiding disclosure can damage the doctor-patient relationship and may be seen as withholding information.
-
Reporting to the ethics committee is unnecessary unless there was negligence or harm.
Ruled-Out Options
-
A. Do not tell the patient what happened
-
Withholding information undermines trust and violates principles of informed consent and autonomy.
-
-
C. Report the incident to the hospital ethics committee
-
This is not warranted if the situation was handled appropriately and no complications occurred.
-
Clinical Tip
After unexpected intraoperative events, disclose honestly and empathetically to patients.
This fosters trust, reduces anxiety, and respects patient autonomy.
Remember the “disclosure triad”: Truthfulness, Compassion, and Reassurance.
Q.173. A 7-year-old child with asthma often misses doses of inhaled medications, leading to poor symptom control. What is the most important step to improve compliance and ensure effective asthma management?
Correct Answer : B
Correct Answer: B. Provide a written asthma action plan
Explanation
Improving adherence in pediatric asthma requires a multifaceted approach:
-
Providing a written asthma action plan is a proven strategy that:
-
Clearly outlines daily treatment, symptom recognition, and when to seek help
-
Empowers both the child and parents for self-management
-
-
Listening and addressing concerns is important but must be paired with concrete tools like the action plan.
-
Sending a nurse for compliance checks is resource-intensive and less practical.
-
Increasing dose without adherence does not improve control and may increase side effects.
Other adherence strategies include simplifying regimens (once-daily dosing), using child-friendly medication forms, and regular follow-up.
Ruled-Out Options
-
A. Listen to the child and address concerns
-
Important but insufficient alone to ensure adherence. Needs to be combined with action plans and education.
-
-
C. Send nurse to check compliance
-
May help in select cases but not practical or sustainable for routine management.
-
-
D. Increase medication dose
-
Does not address the root cause of poor compliance and risks side effects.
-
Clinical Tip
To improve asthma control in children, provide a clear, written asthma action plan and engage both the child and parents.
Simplify drug regimens and use child-friendly medications.
Regular education and follow-up are key to improving adherence.
Q.174. An elderly man with end-stage lung cancer has a Do Not Resuscitate (DNR) order documented by the intensivist. The surgeon plans a debulking surgery that is unlikely to significantly improve quality of life or prognosis. What is the most appropriate next step before proceeding?
Correct Answer : B
Correct Answer: B. Obtain high-risk informed consent from the patient and/or family
Explanation
When planning surgery in a patient with advanced terminal illness and a DNR order:
-
Obtaining a thorough, high-risk informed consent is critical. This involves explaining:
-
The limited benefit and high risks of surgery
-
The impact of the DNR status
-
Possible postoperative complications and outcomes
-
-
Consent must be explicit, documented, and involve the patient and/or family as appropriate.
While involving family (Option C) is important, it is part of the informed consent process rather than a separate step.
Referring to the ethics committee (Option D) may be necessary if there is disagreement or conflict but is not the immediate step.
Changing anesthetist (Option A) does not address the core ethical and clinical issues.
Ruled-Out Options
-
A. Go ahead with surgery with another anesthetist
-
Changing anesthetists does not resolve consent or ethical concerns.
-
-
C. Involve the family in the decision-making process
-
Important but part of the consent process; alone it is insufficient.
-
-
D. Involve hospital ethics committee
-
May be needed if conflict arises but not the immediate next step.
-
Clinical Tip
For high-risk surgery in terminally ill patients, always provide comprehensive informed consent, discussing prognosis, risks, and realistic benefits.
Respect existing DNR orders and align surgical plans accordingly.
Q.175. A mother with confirmed intrauterine fetal demise (IUFD) is counselled before delivery of a stillborn baby. After delivery, the mother insists that the stillborn infant be admitted to the NICU. What psychological reaction is the mother most likely exhibiting?
Correct Answer : A
Correct Answer: A. Denial
Explanation
Denial is a common initial psychological defense mechanism in response to traumatic loss such as IUFD. It helps the mother temporarily avoid the emotional pain of the loss by refusing to accept the reality — in this case, insisting the baby is alive and needs NICU care.
Other defense mechanisms can occur, but denial is the most typical immediate reaction to fetal death.
Ruled-Out Options
-
B. Acting out
-
Involves expressing unconscious feelings through actions (e.g., aggression), not typically the refusal to accept death.
-
-
C. Neglect
-
Refers to failure to provide care, which does not apply here.
-
-
D. Regression
-
Reverting to earlier developmental behaviors, which is less specific here.
-
Clinical Tip
After IUFD, parents may exhibit denial as part of normal grief.
Provide compassionate support, clear information, and allow time for acceptance.
Referral to counseling or bereavement support services can aid psychological adjustment.
Q.176. A patient diagnosed with Ebola virus disease insists on leaving the hospital against medical advice, threatening to discharge themselves. What is the most appropriate immediate action?
Correct Answer : C
Correct Answer: C. Call hospital security to prevent unauthorized discharge
Explanation
Ebola virus is highly contagious and life-threatening, with significant public health implications.
-
Allowing an infected patient to leave the hospital freely risks community spread and outbreak.
-
Hospital security should be called immediately to prevent the patient’s unauthorized discharge and ensure compliance with infection control protocols.
-
Signing DAMA is inadequate because it does not eliminate the risk to the public.
-
Transferring the patient to isolation is appropriate for infection control but does not address the immediate risk of the patient leaving prematurely.
Ruled-Out Options
-
A. Sign DAMA
-
In infectious diseases with public health risk, DAMA is not sufficient to prevent harm to others.
-
-
B. Send to infectious unit
-
Appropriate for care, but does not prevent patient from leaving if they insist on discharge.
-
Clinical Tip
In highly contagious infections like Ebola, patient autonomy may be overridden to protect public health.
Immediate action to prevent unauthorized discharge is essential.
Coordinate with infectious disease specialists and public health authorities promptly.
Q.177. A 42-year-old woman presents with heavy uterine bleeding. Ultrasound shows endometrial hyperplasia. You decide to perform an endometrial biopsy. What is the most appropriate immediate step before proceeding?
Correct Answer : A
Correct Answer: A. Obtain verbal informed consent from the patient
Explanation
-
Informed consent is a mandatory step before any invasive procedure, including endometrial biopsy.
-
Verbal consent is acceptable in many outpatient procedures if the patient is competent and the risks are explained clearly.
-
Calling anesthesia or preparing without consent is unnecessary for a standard office biopsy and violates patient autonomy.
Ruled-Out Options
-
B. Call the anesthesiologist
-
Most endometrial biopsies are done without sedation; anesthesia is not routinely needed.
-
-
C. Prepare the patient without consent
-
Performing procedures without consent is unethical and legally wrong.
-
Clinical Tip
Always obtain informed consent before performing invasive diagnostic procedures.
Explain the purpose, benefits, risks, and alternatives to the patient clearly.
Q.178. A young child consistently refuses to walk with his mother and frequently throws his toys during tantrums. What is the best approach to manage this behavior?
Correct Answer : A
Correct Answer: A. Use time-out as a disciplinary technique
Explanation
-
Time-out is an effective and evidence-based behavioral management strategy for young children exhibiting challenging behaviors.
-
It involves briefly removing the child from the environment where misbehavior occurs, helping them calm down and understand limits.
-
Shouting or physical punishment is not recommended as it can increase aggression, anxiety, and damage the parent-child relationship.
Ruled-Out Options
-
B. Verbal warning and shouting
-
Shouting can escalate the behavior and is not a constructive disciplinary method.
-
-
C. Spanking or physical punishment
-
Physically punishing children is associated with negative psychological outcomes and is discouraged by pediatric guidelines.
-
Clinical Tip
Use time-out consistently for managing disruptive behaviors in young children.
Combine it with positive reinforcement for good behavior to improve compliance and emotional regulation.
Q.179. What are the four key principles for effective reporting in suspected child abuse cases?
Correct Answer : A
Correct Answer: A. Observe, Document, Report, Self-protection
Explanation
Effective child abuse reporting involves these critical steps:
-
Observe: Recognize signs and symptoms of abuse carefully.
-
Document: Accurately record findings with dates, descriptions, and objective details.
-
Report: Notify the appropriate authorities or child protection services promptly.
-
Self-protection: Ensure your own safety and legal protection by following institutional protocols.
This systematic approach ensures timely intervention while safeguarding the reporter.
Ruled-Out Options
-
B. Observe, Document, Referrals, Follow-up
-
While referrals and follow-up are important, initial reporting and self-protection are essential pillars.
-
-
C. Document, Confirm, Report, Conference parents
-
Confirmation and discussing with parents may not always be safe or appropriate, especially if abuse is suspected.
-
Clinical Tip
When suspecting child abuse, always observe carefully, document meticulously, report promptly, and protect yourself legally by following institutional protocols.
Q.180. A patient has recently experienced an abortion. What is the most appropriate next step in managing her care?
Correct Answer : B
Correct Answer: B. Show empathy and explore her emotions
Explanation
-
After an abortion, emotional support and exploration of feelings are critical to help the patient process grief, guilt, or other complex emotions.
-
Empathy combined with open-ended questions facilitates psychological healing and trust.
-
While medical management is important, addressing emotional well-being should come first.
-
Neglecting emotional care can delay recovery and worsen mental health outcomes.
Ruled-Out Option
-
A. Show empathy and talk about management
-
Discussing medical management alone is insufficient without addressing emotional needs first.
-
Clinical Tip
Prioritize emotional exploration and empathetic listening after abortion to identify and address psychological needs early.
Referral for counseling may be beneficial if distress persists.
Q.181. A 40-year-old woman at 24 weeks gestation had a fever for several days but did not seek medical care. What is the most likely reason for her delay?
Correct Answer : B
Correct Answer: B. Neglect
Explanation
-
Neglect refers to failure to seek timely medical care despite symptoms, often due to lack of awareness, access, or personal oversight.
-
Fear of physicians (Option A) can contribute but is less common compared to neglect in many settings.
-
Identifying neglect is important to provide appropriate counseling and support for prenatal care compliance.
Ruled-Out Option
-
A. Afraid from physician
-
While fear can delay care, neglect due to other social or psychological reasons is more typical.
-
Clinical Tip
Address barriers to care such as lack of knowledge, access, or support to reduce neglect in pregnant patients.
Early antenatal education improves timely presentation for symptoms.
Q.182. Which of the following questions is more related to the assessment of alcohol addiction severity?
Correct Answer : A
Correct Answer: A. Do you feel guilty because you are alcoholic?
Explanation
-
The question about feeling guilty reflects recognition of the negative consequences of alcohol use, which is a key feature assessed in addiction severity and is part of the CAGE questionnaire (“G” = Guilt).
-
Asking if the patient thinks they are alcoholic assesses insight but is less specific to addiction severity or impact.
Ruled-Out Option
-
B. Do you think you are alcoholic?
-
More about self-awareness or insight rather than severity of addiction.
-
Clinical Tip
Use validated tools like the CAGE questionnaire to assess alcohol addiction quickly.
Questions about guilt and consequences help evaluate the severity and readiness to change.
Q.183. A doctor instructs a nurse to move a bedridden patient, but the nurse forgets to inform the rest of the healthcare team. What is the most appropriate description of this error?
Correct Answer : A
Correct Answer: A. Poor communication
Explanation
-
Failure to relay important information between healthcare team members constitutes poor communication.
-
This can lead to errors, compromised patient safety, and care coordination issues.
-
It is distinct from negligence (which implies breach of duty), lack of knowledge, or insubordination.
Ruled-Out Options
-
B. Negligence
-
Would require proof of breach of duty causing harm.
-
-
C. Lack of knowledge
-
Not applicable if nurse understood but forgot to communicate.
-
-
D. Insubordination
-
Refusal to obey orders, which is not the case here.
-
Clinical Tip
Implement standardized communication tools like SBAR (Situation, Background, Assessment, Recommendation) to reduce errors.
Encourage team briefings and check-backs to confirm information transfer.
Q.184. A couple is diagnosed with infertility due to tubal blockage in the female partner. To whom should this information be communicated?
Correct Answer : C
Correct Answer: C. Both husband and wife
Explanation
-
Infertility affects both partners emotionally and medically; therefore, disclosure should involve both husband and wife.
-
This respects patient confidentiality while promoting shared decision-making and support.
-
Communicating with both partners fosters trust, joint responsibility, and collaborative management.
Ruled-Out Options
-
A. Tell wife only
-
Excludes the husband from important information impacting the couple.
-
-
B. Tell husband only
-
Ignores the wife's direct involvement and right to know.
-
Clinical Tip
Always involve both partners in infertility counseling and management discussions.
Ensure privacy and sensitivity while promoting teamwork.
Q.185. A mother who is a physician requests a Do Not Resuscitate (DNR) order for her critically ill child. What is the most appropriate action?
Correct Answer : A
Correct Answer: A. Respect her wishes and initiate DNR order
Explanation
-
Parental decisions, including those of physician-parents, must be respected when they are in the best interest of the child and align with medical ethics and legal frameworks.
-
DNR orders require careful discussion, informed consent, and agreement with the medical team and parents.
-
Ignoring parental wishes without due process violates ethical principles and may cause distress.
Ruled-Out Option
-
B. Ignore and intubate
-
Overrides parental autonomy and may cause ethical and legal issues unless the child’s best interest demands intervention.
-
Clinical Tip
When a parent requests a DNR, hold multidisciplinary discussions, clarify prognosis, ensure understanding, and document decisions carefully.
Ethical committees can assist in complex cases.
Q.186. A preterm baby (>25 weeks gestation) with congenital anomalies requires resuscitation at birth, but the mother refuses consent, insisting no intervention. What is the most appropriate action?
Correct Answer : B
Correct Answer: B. Override refusal and proceed with resuscitation and intubation
Explanation
-
For babies ≥25 weeks gestation, there is a reasonable chance of survival with intensive care.
-
In cases where the mother refuses resuscitation but the newborn’s survival is probable and resuscitation is medically indicated, healthcare providers have a duty to act in the best interest of the infant.
-
Ethical and legal principles often permit overriding parental refusal if it places the child at significant risk of preventable death or disability.
-
For extremely preterm infants (<24 weeks), decisions may be more nuanced, and respecting parental wishes is more common due to poor prognosis.
Ruled-Out Option
-
A. Respect the mother’s refusal
-
Acceptable only if prognosis is extremely poor or gestational age is borderline viability.
-
Clinical Tip
When parental refusal conflicts with the newborn’s best interest, seek ethics consultation and hospital legal advice.
Gestational age and prognosis guide resuscitation decisions.
Q.187. A preterm baby with acute life support (ALS) indications needs intubation, but the family refuses consent. What is the appropriate next step?
Correct Answer : B
Correct Answer: B. Override refusal and proceed with intubation
Explanation
-
In critically ill neonates with a good chance of survival and benefit from intervention, the medical team has an ethical duty to act in the best interest of the child, even if the family refuses consent.
-
Life-saving interventions such as intubation should not be withheld if the prognosis is reasonable.
-
This approach aligns with pediatric ethical guidelines and legal standards in many countries.
Ruled-Out Option
-
A. Respect the family’s refusal
-
Generally inappropriate when withholding treatment would likely result in preventable death.
-
Clinical Tip
Engage in clear communication with family, seek ethics and legal consultation if refusal persists.
Prioritize the infant’s best interest in urgent, life-threatening situations.
Q.188. A 7-year-old girl appears underweight and neglected. When asked, the mother complains that she is busy caring for younger children. What is the best next step?
Correct Answer : A
Correct Answer: A. Contact a social worker
Explanation
-
Child neglect requires multidisciplinary intervention, including social services to assess family support and provide necessary assistance.
-
Social workers can help address environmental and caregiving issues leading to neglect.
-
Psychiatric referral for the mother is not the first step unless mental illness is suspected.
-
Nutritional supplements alone do not address the underlying social problems.
Ruled-Out Options
-
B. Refer mother to psychiatrist
-
Only if mental illness is evident or suspected.
-
-
C. Provide nutritional supplements only
-
Addresses symptoms, not cause.
-
-
D. Ignore the situation
-
Neglect must not be ignored; this is harmful.
-
Clinical Tip
Always assess for neglect signs and involve social services early to provide family support and protect the child.
Document findings thoroughly.
Q.189. A female patient comes to the ER complaining that her neighbor abused her. She continues to talk about him even though he is not present. What is the most likely diagnosis?
Correct Answer : A
Correct Answer: A. Delusions and hallucinations
Explanation
-
Persistent belief of being abused by someone who is not present may indicate delusions (fixed false beliefs) and possibly auditory hallucinations (hearing voices).
-
This is typical of psychotic disorders such as schizophrenia or severe mood disorders with psychosis.
-
Anxiety and adjustment disorders involve distress but usually not fixed false beliefs.
-
Factitious disorder involves intentionally producing symptoms, not necessarily hallucinations or delusions.
Ruled-Out Options
-
B. Anxiety disorder
-
Involves worry, not fixed false beliefs or hallucinations.
-
-
C. Adjustment disorder
-
Emotional response to stress without psychosis.
-
-
D. Factitious disorder
-
Deliberate symptom fabrication, not psychosis.
-
Clinical Tip
Distinguish between psychotic symptoms and normal distress by assessing reality testing and presence of hallucinations.
Early psychiatric evaluation is essential.
Q.190. Smoking reduces life expectancy by approximately how many years?
Correct Answer : B
Correct Answer: B. 10 years
Explanation
-
Smoking is a leading cause of preventable death worldwide.
-
Studies estimate that smokers lose on average about 10 years of life expectancy compared to non-smokers.
-
This reduction is due to increased risk of cardiovascular disease, cancers, respiratory illnesses, and other health problems.
Ruled-Out Options
-
A. 5 years — Underestimates the impact.
-
C. 15 years — Slightly overestimates average reduction.
-
D. 20 years — Overestimates typical life expectancy loss for most smokers.
Clinical Tip
Encourage smoking cessation at any age to improve health outcomes and increase life expectancy.
Even quitting later in life yields significant benefits.
Q.191. A female patient arrives at the ER with bruises on her hand, accompanied by police. She appears sad and hesitant to speak. What is the most appropriate initial approach?
Correct Answer : A
Correct Answer: A. Introduce yourself calmly, ensure privacy, and ask open-ended questions about her safety
Explanation
-
When abuse is suspected, building trust and providing a private, safe environment are crucial for accurate history and patient support.
-
Avoid involving family or police in the initial history-taking as this may inhibit disclosure.
-
Physical exam is important but should be combined with sensitive history taking.
-
A broad smile may be perceived as inappropriate or insensitive in such a distressing situation.
Ruled-Out Options
-
B. Take history only in presence of family or police
-
May inhibit honest communication and compromise confidentiality.
-
-
C. Focus immediately on physical examination
-
Neglects the importance of empathetic communication first.
-
-
D. Smile broadly and start with chief complaint
-
May appear insensitive to patient’s emotional state.
-
Clinical Tip
Always provide privacy and ensure the patient feels safe. Use empathetic, nonjudgmental communication when abuse is suspected.
Document findings meticulously and involve social or legal services as needed.
Q.192. A father is angry because his daughter insists on joining a gym. What is the most appropriate response?
Correct Answer : A
Correct Answer: A. Explore why he is angry
Explanation
-
Exploring the father's concerns helps understand underlying issues (e.g., cultural beliefs, safety fears) and promotes effective communication.
-
Simply agreeing without understanding may alienate the family.
-
Ignoring feelings or dismissing concerns can increase conflict.
-
Advising the daughter to stop without dialogue may affect autonomy and trust.
Ruled-Out Options
-
B. Agree immediately with the father
-
Avoids understanding the real issue and may worsen conflict.
-
-
C. Ignore the father’s feelings
-
Disrespectful and may harm family dynamics.
-
-
D. Advise daughter to stop insisting
-
Limits patient autonomy without addressing family concerns.
-
Clinical Tip
Always explore family members' emotions and perspectives before giving advice to facilitate mutual understanding and support.
Q.193. An 8-year-old child is brought in looking unkempt with signs of failure to thrive. The mother expresses anxiety about caring for her other children. What is the most appropriate next step?
Correct Answer : B
Correct Answer: B. Refer to social worker
Explanation
-
Signs of neglect (poor hygiene, failure to thrive) and maternal anxiety about caregiving suggest family and social support issues.
-
Referral to a social worker helps assess the home environment, provide support, and connect the family with resources.
-
Psychological referral may be useful later but social evaluation comes first.
-
Immediate medical treatment is important but addressing social causes is essential for long-term care.
-
Advising to return later delays necessary intervention.
Ruled-Out Options
-
A. Refer to psychology
-
Psychological support is helpful but does not address immediate social needs and family environment.
-
-
C. Start nutritional supplements immediately
-
Important but incomplete without social support.
-
-
D. Advise mother to rest and return later
-
Delays essential intervention.
-
Clinical Tip
Always assess social determinants in cases of child neglect and involve social services early to provide holistic care.
Q.194. An 80-year-old patient undergoing treatment for tuberculosis develops acute liver failure and is admitted to the ICU. The patient's son arrives, very angry and upset. What is the most appropriate initial response by the physician?
Correct Answer : C
Answer:
C) Calm the son and explain that further investigations are necessary to understand the situation.
Explanation:
In critical care, communication with family members, especially in emotionally charged situations, is essential. The best approach is to first calm the family member, acknowledge their concerns, and explain the plan clearly — here, that further investigations are needed to assess the cause and extent of liver failure.
-
Option A is inappropriate as a firm, dismissive tone may escalate anger and mistrust.
-
Option B might seem reassuring but can appear dismissive of the family's distress and does not address their immediate emotional state.
-
Option D avoids transparency and can increase anxiety and suspicion.
Effective communication should combine empathy, clarity, and honesty to maintain trust and cooperation.
Clinical Tip:
In ICU settings, when families react emotionally, prioritize active listening and de-escalation. Use calm tone, acknowledge feelings, and explain next steps clearly. This improves cooperation and supports shared decision-making.
Q.195. Which of the following characteristics is most important for an effective screening tool?
Correct Answer : A
Answer:
A) Easy and fast to perform
Explanation:
A good screening tool should be easy and quick to perform so that it can be applied broadly and efficiently in large populations. Screening aims to detect disease early in asymptomatic individuals, so simplicity and speed increase compliance and feasibility.
-
Option B: Screening is generally more useful for common diseases with significant public health impact, not rare diseases, because the positive predictive value decreases with rarity.
-
Option C: Screening tools for diseases with unknown pathophysiology are difficult to design and generally lack validity; moreover, screening prioritizes sensitivity over specificity.
-
Option D: Screening tools must have high sensitivity to minimize false negatives; low sensitivity would miss many cases.
Clinical Tip:
Remember the Wilson and Jungner criteria for screening programs: the condition should be common enough, have a recognizable latent phase, and the screening test must be simple, safe, sensitive, and acceptable.
Q.196. A patient with spinal muscular atrophy (SMA) develops respiratory muscle fatigue requiring intubation. The mother refuses consent because one of her other children with SMA previously died despite similar interventions. What is the most appropriate next step?
Correct Answer : B
Answer:
B) Proceed with intubation to provide life-saving treatment
Explanation:
In an emergency where the patient is unable to protect their airway or maintain adequate ventilation, life-saving treatment takes priority. Intubation should be performed promptly even if family members refuse consent, especially if the patient’s life is at immediate risk.
-
Option A is not appropriate as the urgent nature of respiratory failure demands immediate intervention. Ethical consultation can follow once the patient is stabilized.
-
In emergencies, consent may be implied if delay risks death or serious harm.
Clinical Tip:
In acute life-threatening emergencies, physicians are ethically and legally justified to perform necessary interventions without prior consent if delay would endanger the patient’s life. Always explain the situation to family as soon as feasible.
Q.197. A doctor instructs a nurse to move a bedridden patient. The nurse forgets to inform the other staff involved in patient care. What is this an example of?
Correct Answer : A
Answer:
A) Poor communication
Explanation:
This situation illustrates poor communication within the healthcare team, which can lead to errors or compromised patient safety. Effective communication and handoff between staff are critical, especially for vulnerable patients.
-
Option B (Inadequate staffing) is unrelated to the failure to communicate instructions.
-
Option C (Patient negligence) incorrectly assigns fault to the patient, which is not applicable here.
-
Option D (Medical malpractice) implies a breach of professional duty causing harm, which requires more than just a communication lapse.
Clinical Tip:
Use standardized communication tools like SBAR (Situation, Background, Assessment, Recommendation) to improve clarity and reduce errors in clinical handoffs.
Q.198. During surgery, a resident observes that the consultant performs an unnecessary intervention, but the patient remains unharmed. What should the resident do?
Correct Answer : C
Answer:
C) Document the details accurately in the surgical report
Explanation:
The resident’s professional responsibility is to accurately document all aspects of the surgery in the operative report, including any deviations or unexpected interventions. Transparent documentation is crucial for medical records, continuity of care, and legal protection.
-
Option A: Directly telling the patient without proper process may breach protocol and create mistrust without institutional support.
-
Option B: Reporting to administration may be necessary if harm occurs or repeated errors happen, but initial step is accurate documentation.
-
Option D: Ignoring the event is unprofessional and could have medico-legal consequences.
Clinical Tip:
Always maintain detailed, objective, and factual operative notes. This is essential for patient safety, quality improvement, and medico-legal reasons. If the unnecessary intervention is repeated or causes harm, follow institutional reporting procedures.
Q.199. A doctor instructs a nurse to administer a medication intravenously. The nurse mistakenly gives lidocaine instead of the intended drug because it was placed beside it. The patient subsequently dies. What best describes this situation?
Correct Answer : C
Answer:
C) Disconnection between doctor and nurse
Explanation:
This case exemplifies a communication breakdown (disconnection) between the doctor and nurse regarding medication administration, leading to a fatal error. Proper communication and double-checking protocols are essential to prevent such errors.
-
Option A: Incorrect, as the medicines are different.
-
Option B: The prescription was presumably correct; the error happened during administration, so it’s primarily an administration error rather than a prescription error.
-
Option D: This option is vague and does not capture the core issue, which is communication failure.
Clinical Tip:
To prevent medication errors, always use "read-back" or "repeat-back" communication, clear labeling, and the “five rights” of medication administration: right patient, right drug, right dose, right route, and right time.
Q.200. When you need to break bad news to a patient, what is the best initial approach?
Correct Answer : B
Answer:
B) Ask the patient if they want their family present during the discussion
Explanation:
Breaking bad news requires sensitivity to the patient's preferences. Asking the patient whether they want their family involved respects their autonomy and privacy and allows for a supportive environment tailored to their needs.
-
Option A: Telling family without the patient’s consent can violate patient confidentiality and autonomy.
-
Option C: Although empathy is essential, involving family should be patient-directed.
Clinical Tip:
Use protocols like SPIKES to break bad news:
-
Setting
-
Perception
-
Invitation (ask patient if they want family involved)
-
Knowledge
-
Emotions
-
Strategy and Summary
Q.201. A female patient requires a kidney transplant. Her husband consents to the procedure, but the patient refuses. What is the most appropriate next step?
Correct Answer : B
Answer:
B) Respect the patient’s decision and document it by having her sign a Discharge Against Medical Advice (DAMA) form
Explanation:
Informed consent for any medical procedure must be obtained directly from the patient if they have decision-making capacity, regardless of family members’ opinions. The patient has the right to refuse treatment. If she refuses the transplant, her decision must be respected and clearly documented.
-
Option A: The husband’s consent alone is not legally or ethically sufficient; the patient’s autonomy is paramount.
-
Documenting refusal with a DAMA form protects both the patient’s rights and the medical team’s legal standing.
Clinical Tip:
Always verify the patient’s decision-making capacity and ensure informed consent is obtained from the patient themselves. Family input can support but never replace the patient’s own consent.
Q.202. A surgeon performing an operation on a patient with Hepatitis C is accidentally injured by a needle stick. What is the approximate risk of the surgeon acquiring Hepatitis C infection from this exposure?
Correct Answer : B
Answer:
B) 3%
Explanation:
The risk of Hepatitis C virus (HCV) transmission after percutaneous exposure (e.g., needle stick injury) is approximately 0.2% to 1.8%, with some sources citing up to 3%. The CDC’s data (2010) estimates the risk around 1.8% but ranges vary depending on factors like viral load and injury type.
-
Option A (30%) is too high and more consistent with Hepatitis B in unvaccinated individuals.
-
Option C (0.3%) and Option D (0.03%) underestimate the risk.
Clinical Tip:
After needlestick injury with HCV exposure, no post-exposure prophylaxis is currently recommended, but follow-up testing and counseling are essential. Hepatitis C transmission risk is lower than Hepatitis B but higher than HIV in similar exposures.
Q.203. In a low-prevalence community, a positive result from a screening test is most likely to be:
Correct Answer : C
Answer:
C) False positive
Explanation:
In populations with low disease prevalence, even screening tests with high specificity can yield a higher proportion of false positive results due to the low pre-test probability of disease. This is explained by the effect of Bayes' theorem on positive predictive value (PPV).
-
True positives are less likely because fewer individuals actually have the disease.
-
False positives increase because the number of healthy individuals is large, and even a small false positive rate leads to many false positives.
Clinical Tip:
Understand that screening tests perform differently depending on disease prevalence. Positive predictive value (probability that a positive test is true) decreases as prevalence decreases, increasing false positives.
Q.204. A newborn with an inguinal hernia requires urgent surgical repair. The parents have not given consent. What is the appropriate next step?
Correct Answer : B
Answer:
B) Seek approval from the hospital ethical committee before surgery
Explanation:
For non-life-threatening but urgent procedures in minors where parental consent is not obtained, the physician should seek guidance from the hospital’s ethical committee or legal authority before proceeding. This ensures that the child’s best interest is protected while respecting legal and ethical standards.
-
Option A is only justifiable in life-threatening emergencies where delay would cause harm.
-
Ethical committees provide oversight for decisions involving minors when consent is refused or unavailable.
Clinical Tip:
In pediatric cases, always prioritize obtaining informed consent from guardians. If refused and delay risks harm, escalate to institutional ethical/legal authorities before proceeding.
Q.205. During a CPR session led by a doctor, what is the doctor’s most important role in ensuring patient safety and effective resuscitation?
Correct Answer : D
Answer:
D) Enhance communication among the resuscitation team
Explanation:
Effective communication is crucial during resuscitation to coordinate tasks, reduce errors, and improve patient outcomes. The team leader (doctor) must facilitate clear, concise communication and ensure all team members understand their roles.
-
Options A and B: Delegating is part of leadership but replacing oneself during CPR is generally inappropriate without proper handover.
-
Option C: Practice is important but not an immediate action during active CPR.
Clinical Tip:
Use structured communication tools like closed-loop communication during CPR to confirm instructions and reduce errors.
Choose a Question
×Choose a Question
×
Trial Access Limit Reached
You’ve reached the limit of free content. Subscribe to continue learning without restrictions.